You are on page 1of 107

www.byjusexamprep.

com

Mock Test Solutions in English

Questions

1. Direction: Read the given passage carefully and answer the questions that follow.

Wicker baskets filled with fruit that have survived from the 4th century BC and hundreds of ancient
ceramic artefacts and bronze treasures have been discovered in the submerged ruins of the near-
legendary city of Thonis-Heracleion off the coast of Egypt. They have lain untouched since the city
disappeared beneath the waves in the second century BC, then sunk further in the eight century AD,
following cataclysmic natural disasters, including an earthquake and tidal waves.

Thonis-Heracleion – the city’s Egyptian and Greek names – was for centuries Egypt’s largest port on the
Mediterranean before Alexander the Great founded Alexandria in 331BC.

But the vast site in Aboukir Bay near Alexandria was forgotten until its re-discovery by the French marine
archaeologist Franck Goddio two decades ago, in one of the greatest archaeological finds of recent times.
Colossal statues were among treasures from an opulent civilisation frozen in time. Some of the
discoveries were shown in a major exhibition at the British Museum in 2016

Goddio has been taken aback by the latest discoveries, terming the fruit baskets as “incredible”, having
been untouched for more than 2,000 years.

They were still filled with doum, the fruit of an African palm tree that was sacred for the ancient Egyptians,
as well as grape-seeds.

“Nothing was disturbed,” he said. “It was very striking to see baskets of fruits.”

One explanation for their survival may be that they were placed within an underground room, Goddio said,
noting a possible funerary connotation.

It is within an area where Goddio and his team of archaeologists have discovered a sizeable tumulus (a
www.byjusexamprep.com

mound raised over graves) – about 60 metres long by 8 metres wide – and sumptuous Greek funerary
offerings.

They date from the early fourth century BC when Greek merchants and mercenaries lived in Thonis-
Heracleion. The city controlled the entrance to Egypt at the mouth of the Canopic branch of the Nile. The
Greeks were allowed to settle there during the late Pharaonic period, constructing their own sanctuaries.

Source: https://www.theguardian.com/science/2021/aug/02/fruit-baskets-from-fourth-century-bc-found-in-
ruins-of-thonis-heracleion

From the passage it can be inferred that


B. The discovery of the fruit-filled wicker basket will
A. Traces of Thonis-Heracleion were removed during
shed light upon the funerary practices of the
the colonial occupation of Egypt.
erstwhile Greek civilisation.
D. The discovery of the fruit-filled wicker basket will
C. The excavations reveal that Thonis-Heracleion
shed light upon the funerary practices of the ancient
was a poor city, despite being a large port.
Roman civilisation.
2. It is evident that Thonis-Heracleion was obliviated from human memory and is only now being
rediscovered because
B. While establishing Alexandria as the major port
A. A great fire had torn through the city, and the city, Alexander the Great ensured that Thonis-
burnt remnants of the artifacts have recently washed Heracleion was obliviated from human memory to
ashore the Mediterranean Coasts. prevent it from competing with the grandeur of
Alexandria.
D. The colonisers had archived the priceless artifacts
C. Earthquakes and tidal waves had submerged the
from the excavated city to keep them hidden from
city by the eighth century BC
the rest of the world.
3. The author’s main purpose behind writing the passage is to
A. Inform the readers of the latest findings related to B. Dissuade the readers from visiting the site of
the excavations at the site of the ancient city of Thonis-Heracleion since it has become a large
Thonis-Heracleion sinkhole following excavations
C. Inform the readers of the foul play involved in the D. Criticise the archaeologists of damaging the site
submergence of Thonis-Heracleion in the fourth of the ancient city of Thonis-Heracleion.
www.byjusexamprep.com

century BC
4. What made Thonis-Heracleion one of the most important cities of Ancient Egypt?
A. It was one of the largest ports, and was B. It was the seat of knowledge of the Ancient
strategically situated at the heart of Europe, with Egyptians, and had a library bigger than the one in
access to all major European cities. Alexandria.
C. It was the centre of the spice and silk trade in D. It was the one of the largest ports on the
Africa. Mediterranean and controlled the entrance to Egypt.
5. The word ‘sumptuous’ is the antonym of
A. Luxurious B. Opulent
C. Lavish D. Cheap
6. Direction: Read the given passage carefully and answer the questions that follow.

More than a billion sea creatures across the Pacific north-west perished in this year’s heatwave. And it’s
just a taste of what’s to come When forecasts foreshadowed the Pacific north-west’s devastating
heatwave at the end of June, marine biologist Christopher Harley was alarmed and intrigued.

Then came the smell, and his feelings somberly shifted. “It was this putrid smell of decay,” Harley said.
Across hundreds of miles of coastline, the extreme heat baked the barnacles, seaweed, and small sea
creatures exposed to the elements along the shore. Starfish that failed to crawl to shadier spots were
cooked alive. Mussels laid agape along the rocks, the tissue crisped between their shells.

Armed with special equipment, Harley and his team of researchers descended to the beaches to measure
the body temperatures of the mussels, but it was too late – by the third day of the record-breaking
heatwave they all had died. “We were just walking across carpets of dead mussels on the shore in awe,”
he said.

Initial estimates show that over a billion creatures that live in the shallow waters across the Pacific north-
west perished in the heat that week. Scientists expect the impact will have a trickle-down effect on the
ecosystem and the other animals that rely on those that died for food and habitat.

The unprecedented heat events that scorched the west this summer – smashing dozens of local
temperature records across the region – had disastrous effects on people, plants, and animals. Climate
www.byjusexamprep.com

scientists say this is just a taste of what’s to come.

As the environment continues to warm due to human-caused global heating, spiking temperatures will
become more frequent, more intense, and last longer. Because heat and drought are inextricably linked,
the compounding catastrophes that have plagued the west this summer will persist into the future,
continuing to wreak havoc on ecosystems, infrastructure, and agriculture.

Scientists are still working to document and understand the impact of the recent heatwaves. But the
research has been clear that this anomalous event could become the norm over the next 30 years as the
planet continues to warm.

Source: https://www.theguardian.com/us-news/2021/jul/31/global-heating-climate-crisis-animals-water-
crops

Which of the following is the author trying to suggest by citing the latest research on the impact of the
recent heatwaves?
A. The seriousness of the environmental cost of B. The lack of scientific basis linking climate change
human-caused global heating. to the heatwaves sweeping over continents.
C. The change in the aquatic ecosystem due to D. The impact of climate change on natural disasters
receding sea levels. like Tsunamis and earthquakes.
7. Which of the following is not a premise of the author’s argument against the disastrous impact of global
heating?
A. The possibility of the death of over a billion sea
B. Heat and drought, being intwined events, having
creatures across the Pacific North-West having a
disastrous impact on agriculture.
trickle-down impact on the ecosystem.
C. The perishing of the sea creatures due to the
D. The direct impact of global heating on
heatwaves leading to better fishing opportunities in
ecosystems, infrastructure, and agriculture.
the future.
8. Which of the following is not the learning outcome from the heatwaves?
A. Indiscriminate exploitation of the environment will B. Indiscriminate exploitation of the environment will
not have an impact on human life and property. have an adverse impact on human life and property.
C. Exploitation of the environment will wreak havoc D. Global heating is a man-made disaster rather
not simply on human lives, but also on the larger than a natural one.
www.byjusexamprep.com

environment.
9. The expression ‘trickle-down effect’ as used in the passage means
A. Consequential effect B. Adverse effect
C. Positive effect D. Realistic effect
10. The expression ‘exposed to the elements’ as used in the passage means
A. Shielded from harm B. In harm’s way
C. Immune to diseases D. Secure from changes
11. Direction: Read the given passage carefully and answer the questions that follow.

Alternative work arrangements in the gig economy have the potential to absorb more women and increase
their participation in the workforce with some amount of reskilling, according to a latest study.

Gig economy is a labour market characterised by the prevalence of short-term contracts or freelance work
as opposed to permanent jobs. A study on "The Impact of COVID 19 and Industry 4.0 on Future of Work
for Women, An Insight from Formal Sector in India" underscores that alternative work arrangements in the
gig economy have the potential to absorb more women and increase their participation in the workforce.

"A boost in use of digital technology and increased acceptance of virtual working for sales and distribution
jobs, could open opportunities for women to enter fields where interactions are managed through apps
and phone calls," the study states.

"The study indeed gives a hope for a better future with more employment opportunities for women.
However, concerted efforts are needed to understand how new technologies are impacting specific
industries and to address challenges facing women in entering or remaining in the workplace," stated the
UNDP Deputy Resident Representative in India, at the formal launch event of the study.

The survey also reflects that some key sectors that are likely to see an increase in women's employment
are health and pharmaceuticals (by 59 per cent), electrical and electronics (by 44 per cent) and fast-
moving consumer goods (perishable goods) industry (by 41 per cent).

At the same time, women's employment in the finance and accounting divisions may moderately change
due to the adoption of new technology. Around 73 per cent of the respondent firms agree that reskilling,
www.byjusexamprep.com

specifically in the formal sector -- is likely to play a crucial part in shaping the work of tomorrow. About 83
per cent of those who agreed to the skilling proposition also indicate that reskilling will be crucial to absorb
more women in the world of work in India, the study states.

The study on "The Impact of COVID 19 and Industry 4.0 on Future of Work for Women, An Insight from
Formal Sector in India"____________
A. Predicts that gig economy will reduce the B. Predicts that gig economy will increase the
absorption of more women in the workforce. absorption of more women in the workforce.
C. Predicts that gig economy will not have any
D. Predicts that gig economy will increase the
impact on the absorption of more women in the
absorption of more men in the workforce.
workforce.
12. Which of the following can be inferred as a caution by the author?
A. It is crucial to assess the impact of new B. Women are not deterred by new technologies,
technologies on women to be absorbed into the and reskilling should be made optional to save
workforce. resources.
C. Continuance of old technologies will deter the D. The lack of skilled labour market will deter the
ascendence of ‘gig economy’ in developing ascendence of ‘gig economy’ in developing
countries. countries.
13. In the context of the passage, which of the following best describes gig economy?
A. An economy based on permanent induction of B. An economy based on increased foreign
labour into Public Sector Units. investment.
C. An economy marked by the prevalence of a
labour market hired on the basis of short-term D. All the above.
contracts or freelance work.
14. The tone of the passage is
A. Cynical B. Critical
C. Scornful D. Objective
15. According to the passage, which of the following comes as a concern related to gig economy?
A. Lack of educational and health infrastructure. B. Lack of job security
C. The permanent nature of jobs D. None of the above
16. Direction: Read the given passage carefully and answer the questions that follow.
www.byjusexamprep.com

To understand the structure of a protein in detail, scientists have had to make comparatively large
amounts of it, coax those molecules into forming a crystal (or, more recently, flashfreeze them) and
bombard the sample with xrays (or, if it is a frozen one, electrons). These procedures take time, money
and effort. Only a tiny fraction of the proteins whose sequences are known have been studied this way.

Now things look likely to get considerable easier. AlphaFold, a very elegant piece of software developed
by DeepMind, a British AI company, has learned from the detailed study of sequence and structure to
make predictions of protein shapes using just the aminoacid sequences of their very bendy backbones.

However, those suspicious of hype rightly point out, not all of these will be equally good and better
prediction does not do away with the need for other ways of exploring protein structure. AlphaFold will not
be the last word in the application of AI to the problem. All that said, the software looks likely to be
massively useful, helping researchers spot possibilities and dead ends more quickly and letting them take
on projects they would otherwise have steered clear of. If AlphaFold is not soon providing useful
pharmacological results, as well as lots of basic biology, it will not be for want of application.

AlphaFold will also help usher in an era of altogether stranger things. The “potential protein space” the AI
calculations reveal is thus a cosmos unto itself, ordered yet near infinite. The bit that evolution has
explored so far—which contains wonders as diverse as proteins that flex like springs, spin like wheels,
extend like pistons and crank like ratchets, that turn sunlight into chemical energy, that build, demolish and
recycle all the components of life and do more besides—is but the tiniest corner of it. Already some
scientists are working on “de novo” proteins well outside nature’s comfort zone as ways of making tiny
mechanisms and machines. As tools like AlphaFold increase the ease of molecular design, they will be
joined by many more. Compared with the designs, evolution has honed over billions of years such things
as this will be the crudest of toys, at least to begin with. But who can say what new flesh the designers’
words will eventually bring into being?

Source: https://www.economist.com/leaders/2021/07/31/remarkable-progress-has-been-made-in-
understanding-the-folding-of-proteins

The passage can be best termed as?


A. Expository B. Controversial
C. Abstract D. Narrative
www.byjusexamprep.com

17. What is the passage’s main purpose?


A. Warn about the danger of altering the protein B. Advertise about the new software AlphaFold,
structure. developed by DeepMind.
C. State how AI technology like AlphaFold is going D. To counter the pharmaceutical changes that AI
to ease the study of protein. software like AlphaFold will bring.
18. Which of the following is known about protein?
A. The protein can turn sunlight into electrical B. The proteins have a very rigid immovable
energy. structure.
C. The proteins have a flexible structure which can D. The structure of protein is unable to recycle the
be extended. components of life.
19. Only a tiny fraction of the proteins whose sequences are known because
A. AI like AlphaFold were not funded by the B. The procedures of studying the protein structure
government. takes time, money and effort.
C. The scientists were sceptical and uncomfortable D. The pharmaceutical return about studying protein
to explore the infinite protein space. structure is not rewarding.
20. The “potential protein space” the AI calculations reveal is thus a cosmos unto itself

The ____________ in the above sentence refers to the ___________________ nature of protein
structure.
A. Simile-Bounding B. Metaphor-Myriad
C. Onomatopoeia-Secluded D. Anaphora………Oscillating
21. Direction: Read the given passage carefully and answer the questions that follow.

The worse things get, the more a person is compelled to optimize. It is like going to a barre class or eating
lunch at a fast-casual chopped-salad chain, like Sweetgreen, which feels less like a place to eat and more
like a refuelling station. Sweetgreen is a marvel of optimization: a line of 40 people – a texting, shuffling,
eyes-down snake – can be processed in 10 minutes, as customer after customer orders a kale caesar
with chicken without even looking at the other, darker-skinned, hairnet-wearing line of people who are
busy adding chicken to kale caesars as if it were their purpose in life to do so and their customers’
purpose in life to send emails for 16 hours a day with a brief break to snort down a bowl of nutrients that
ward off the unhealthfulness of urban professional living.
www.byjusexamprep.com

The ritualization and neatness of this process (and the fact that Sweetgreen is pretty good) obscure the
intense, circular artifice that defines the type of life it’s meant to fit into. The ideal chopped-salad customer
needs to eat his $12 salad in 10 minutes because he needs the extra time to keep functioning within the
job that allows him to afford a regular $12 salad in the first place. He feels a physical need for this $12
salad, as it’s the most reliable and convenient way to build up a vitamin barrier against the general
malfunction that comes with his salad-requiring-and-enabling job. As Matt Buchanan wrote at the Awl in
2015, the chopped salad is engineered to “free one’s hand and eyes from the task of consuming nutrients,
so that precious attention can be directed toward a small screen, where it is more urgently needed, so it
can consume data: work email or Amazon’s nearly infinite catalogue or Facebook’s actually infinite News
Feed.

On today’s terms, what Buchanan is describing is the good life. It means progress, individuation. It’s what
you do when you’ve gotten ahead a little bit, when you want to get ahead some more. The hamster-wheel
aspect has been self-evident for a long time now. But today, in an economy defined by precarity, more of
what was merely stupid and adaptive has turned stupid and compulsory. Vulnerability, which is ever
present, must be warded off at all costs. It’s very easy, under conditions of artificial but continually
escalating obligation, to find yourself organizing your life around practices you find ridiculous and possibly
indefensible.

Source: https://www.theguardian.com/news/2019/aug/02/athleisure-barre-kale-tyranny-ideal-woman-
labour

Which of the following best describes the purpose of the author of this passage?
A. To state that optimization has become a part and B. The identify the need of optimization to progress
parcel of modern life. in life.
C. To show how places like Sweetgreen teaches us D. To remove the unhealthfulness, a result of urban
about optimization. professional living.
22. The term ‘snort down a bowl of nutrients’ as used in the passage means
A. Eat a healthy meal B. Eat fast for eating sake
C. Eat slow for pleasure D. Eat for the taste of the food
23. What does the author mean by “hamster-wheel aspect” of life?
A. To keep oneself happy and healthy by leading life B. To be busy all the time but never achieving
in a holistic manner. reaching the end of a task.
www.byjusexamprep.com

C. To lead our life driven by purpose and a D. To make our life action oriented and not rewards.
predetermined goal.
24. What according to the author Matt Buchanan is the ‘good life’?
A. Optimize our life in a manner where we can earn
B. Optimize our life in a manner where can be
enough to go places such Sweetgreen without
vulnerable and not worry about tomorrow.
thinking much about the cost.
C. Optimize our life in a manner where working or
consuming data is more important that consuming D. Both A and B
food.
25. Which of the following sentences reflects the reversal of the statement “Vulnerability, which is ever
present, must be warded off at all costs.”
A. We must take part in stupid and adaptive customs B. We must organize our life around practices which
as our life has become precarious. are ridiculous in nature.
C. We must show resilience instead of giving into D. We must keep our selves precarious as it is the
practices which have no sense. cost of progress.
26. Direction: Read the given passage carefully and answer the questions that follow.

It is a shocking, garish sight to come across on a peaceful mountainside. Hike high enough in the French
alps during the late spring and early summer, and there is a good chance that you will come across some
rather strange patches of snow among the grey limestone and stunted clumps of vegetation. This snow
isn't white – it's blood red.

The peculiar phenomenon – sometimes known as blood snow – is the result of a defence mechanism
produced by microscopic algae that grow in the Alpine snow. Normally these microalgae have a green
colour as they contain chlorophyll, the family of pigments produced by most plants to help them absorb
energy from sunlight. However, when the snow algae grow prolifically and are exposed to strong solar
radiation, they produce red-coloured pigment molecules known as carotenoids, which act as a sunshield
to protect their chlorophyll.

While red snow algae have been known for a long time (it is mentioned in a book published in 1819 as
having been discovered during an expedition to the Arctic in 1818) it is still steeped in mysteries that
scientists are attempting to unravel.
www.byjusexamprep.com

But understanding more about red snow algae carries a significance far greater than simply explaining the
existence of strange-coloured patches in the Alps and near the poles. Its appearance and disappearance
are important markers of climate change and how it is affecting the delicate ecosystems where the algae
are found.

According to a professor of interface geochemistry at the German Research Centre for Geosciences in
Potsdam, red snow is becoming more common due to global warming and changing the climatic status
quo. "The rise in the atmospheric carbon dioxide levels increases the temperature, which leads to more
snow melting," she says. "The moment there is liquid water on the snow, the algae start growing."

This increasing abundance of red snow algae may also be contributing to climate change too. The red
pigment turns the snow surface dark, reducing the amount of light and heat it reflects back into space –
something known as the albedo effect. By trapping more of the Sun's heat, the snow melts even faster,
allowing the algae to proliferate further. "There is a runaway effect in which the algae melt their preferred
habitat," says Benning. "It’s as if they are destroying their own house."

Source: https://www.bbc.com/future/article/20210729-why-snow-in-the-alps-is-turning-red

Which of the following best describes the tone of the author?


A. Optimistic B. Anxious
C. Compassionate D. Critical
27. Why does the Alpine snow algae turn red in colour?
A. The Alpine snow algae are green in colour, but B. The Alpine snow algae was recently discovered,
due to the sunlight on them, they seem red from a and scientists are yet to find out why they turn red in
lower altitude. colour.
C. Due to secretion of red-coloured pigment D. The Alpine snow algae are green in colour, but as
molecules carotenoids which acts as a sunshield a result of climate change, they are turning into red
against chlorophyll. in colour.
28. What does ‘garish’ mean in the context of the passage?
A. Drab B. Calamitous
C. Heinous D. Gaudy
29. The ‘status quo’ as used in the passage means
www.byjusexamprep.com

A. Previously popular opinions. B. Already existing conditions.


C. Strategies to counter climate change. D. Following a set agenda for climate change.
30. What does the statement “it’s as if they are destroying their own house” mean in context of the passage?

A. The algae by reflecting sun’s heat are melting B. The algae by releasing their own heat, are melting
their own habitat. their own habitat.
C. The algae by creating a buffer zone between D. The algae by trapping sun’s heat are melting the
snow and sunlight, are melting their own habitat. ice, their own habitat.
31. Direction: Read the given passage carefully and answer the questions given below:
Last year, the Rajasthan government announced its plan to develop the Bundi sanctuary as a tiger reserve
to provide a second habitat for tigers in the Ranthambore Reserve. According to experts, there is
overcrowding at the Ranthambore reserve, which houses 77 tigers. Over 300 sq km area in the proposed
sanctuary will be left as a critical habitat for the animals, while the rest will be a buffer zone, the officials
familiar with the matter said. The NTCA is now expected to send a committee to review the Bundi
sanctuary, a second official said. Tigers from Ranthambore can only be translocated to the new sanctuary
after the NTCA’s approval. Upgrading the sanctuary would resolve the space crunch at Ranthambore
reserve, the second official said. Wildlife expert and founder secretary Sariska Tiger Foundation, Dinesh
Durani, said the move to increase tiger population and reduce territorial infights is welcome. “But the focus
should be on improving the existing tiger habitat and corridors for their easy disbursal,” he said.

Source: Article taken from The Hindustan Times Written by: Sachin Saini, Dated: 05 July, 2021

Which of the following wildlife sanctuaries has recently become the 4th Tiger reserve of Rajasthan ?
A. Kumbhalgarh Wildlife Sanctuary B. Ramgarh Vishdhari wildlife sanctuary
C. Jaisamand Sanctuary D. Mount Abu Wildlife Sanctuary
32. What is the theme for the International Tiger Day 2021 ?
A. 'Save Tiger, Save Biodiversity' B. 'Tigers Lives Matters'
C. 'Their survival is in our hands' D. 'Tiger Conservation for Sustainable Development'
33. During which of the following important meetings, it was decided to celebrate 29th July as Global Tiger
Day across the world ?
A. Vienna Convention B. St. Petersburg declaration
C. UNESCO'S International Convention D. Tokyo Convention
34. Which of the following wildlife sanctuary/national park is located on the tri-junction of Rajasthan, Madhya
www.byjusexamprep.com

Pradesh and Uttar Pradesh ?


A. Keoladeo National Park B. Sajjangarh wildlife sanctuary
C. Desert National Park D. National Chambal Gharial Wildlife Sanctuary
35. In which year was the National Tiger Conservation Authority (NTCA) established as a statutory body which
works under the Ministry of Environment, Forests and Climate Change?
A. 2010 B. 1999
C. 2005 D. 2008
36. Direction: Read the given passage carefully and answer the questions given below:

A Bengaluru-based start-up has received the National Award 2021 from the Technology Development
Board for developing a commercial solution for conversion of carbon dioxide to chemicals and fuels, the
Department of Science and Technology said on Monday.

"It has led to improvisation of process engineering to enhance the production of chemicals and fuels from
anthropogenic CO2 generated from various sources, including coal and natural gas power generation
sectors, steel industry, cement industry, chemical industries; and integrating multiple components involved
in CCUS (Carbon capture, utilization and sequestration) to develop a complete solution for the
environmental issues due to global warming,” the Department of Science and Technology (DST) said.

The research was carried out by Prof Sebastian C Peter and his group from the New Chemistry unit at
JNCASR. He is also the co-founder and director of the start-up which was started from funding by DST’s
Nano Mission. The start-up has also signed an agreement with JNCASR, an autonomous institute of
Department of Science and Technology, for transfer of the technology based on lab-scale research on
reducing CO2 to methanol and other useful chemicals and fuels.

Source: Article taken from Outlook India Dated: 24 May, 2021

Which Bengaluru-based start-up has recently received the National Award for Recycling Carbon
Technology 2021 by the Department of Science and Technology, Government of India ?
A. 'Breathe Applied Sciences' B. 'Mission CO2 to Methanol'
C. 'Mission Clean Carbon' D. 'Clean Carbon Metallurgy'
37. Every year, 11th May is being celebrated as National Technology Day. What is the theme for National
Technology Day 2021 ?
www.byjusexamprep.com

A. 'Post-pandemic Scenario of Science and B. 'Emergence of Science & Technology during


Technology' Pandemic'
C. 'Science and Technology for a Sustainable D. 'Rebooting the economy through Science and
Future' Technology'
38. In terms of Carbon Technology, improvisation of process engineering of Carbon and it's recycling, what
does the term CCUS stand for ?
A. Carbon Capture, Utilization and Sequestration B. Carbon Capture, Ununhexium and Sequencing
C. Carbon Capitalization, Unsaturation and D. Carbon Cancellation, Unsaturation and
Sequestration Sequencing
39. Where was the 6th International Conference on Nanoscience and Nanotechnology (ICONN 2021) held
under the aegis of Nano Mission, Department of Science and Technology (DST), Government of India ?
B. Bhabha Institute of Science and Technology,
A. Tata Institute of Fundamental Research, Mumbai
Kanpur
C. SRM Institute of Science and Technology D. Council of Scientific and Industrial Research
(SRMIST), Kattankulathur (CSIR), New Delhi
40. Which of the following is not correct about the Nano Mission (Nano Science and Technology Mission –
NSTM) ?
A. It was launched by the Government of India in B. A total fund of Rs. 1000 crores has been allocated
2000 under the Department of Science and to this mission by the Ministry of Science and
Technology. Technology.
D. Establishment of R&D in Nanoscience
C. Nanotechnology involves the manipulation of
Applications and establishment of Development
matter on atomic, molecular, and supramolecular
Centre for Nanosciences are the primary objectives
scales.
of Nano Mission.
41. Direction: Read the given passage carefully and answer the questions given below:

The Asian Development Bank (ADB) and the Government of India here today signed a loan to improve
transport connectivity and facilitate industrial development in the Chennai–Kanyakumari Industrial Corridor
(CKIC) in the state of Tamil Nadu.

CKIC is part of India’s East Coast Economic Corridor (ECEC), which stretches from West Bengal to Tamil
Nadu and connects India to the production networks of South, Southeast, and East Asia. ADB is the lead
www.byjusexamprep.com

partner of the Government of India in developing ECEC.

Mr. Rajat Kumar Mishra, Additional Secretary, Department of Economic Affairs, signed agreement for the
Tamil Nadu Industrial Connectivity Project on behalf of the Government of India, while Mr. Takeo Konishi,
Country Director for ADB in India, signed for ADB. “The project is key to providing seamless road
connectivity across industrial clusters, transport gateways and consumption centers, and help reduce
logistics and production costs for CKIC’s targeted industries to boost their competitiveness,” remarked Mr.
Mishra.

Source: Article taken from: pig.gov.in, Dated: 16 June, 2021

What is the amount of loan that Asian Development Bank (ADB) is providing to the Government of India
for the Tamil Nadu Industrial Connectivity Project ?
A. $484 million B. $500 million
C. $550 million D. $350 million
42. Which of the following states are members of India’s East Coast Economic Corridor (ECEC) project ?
A. West Bengal, Chattisgarh, Odisha and Andhra
B. Odisha, Andhra Pradesh, Tamil Nadu and Kerala
Pradesh
C. West Bengal, Odisha, Andhra Pradesh and Tamil D. Andhra Pradesh, Tamil Nadu, Kerala and
Nadu Karnataka
43. Government has approved the development of the five industrial corridor projects across the nation.
Which of the following is not a part of the project ?
A. Delhi-Mumbai Industrial Corridor (DMIC) B. Bengaluru-Mumbai Economic Corridor (BMEC)
C. Amritsar-Kolkata Industrial Corridor (AKIC) D. Allahabad-Haldia Industrial Corridor (AHIC)
44. India has recently signed an agreement extending a USD 100 million Line of Credit (LOC) to Sri Lanka for
projects in the _____________.
A. Arms Acquisition & Defence Cooperation B. Solar Energy Sector
C. Infrastructural Development D. 5G Technology & Cyber Security
45. Which of the following statements is/are not correct regarding the Asian Development Bank (ADB) ?

I. It is a regional development bank established on 19th December 1966 which is headquartered in Manila,
Philippines.

II. It is currently constituted of 58 nations as it's member in which 49 are Asian nation's.
www.byjusexamprep.com

III. Japan and the United States of America (USA) are the largest shareholders (each with 15.6% of total
shares) in ADB.

IV. It's main aim is to promote social and economic development in Asia and the Pacific region.

Mark your answer as:


A. Only I and III B. Only II
C. Only II and III D. Only I and IV
46. Direction: Read the given passage carefully and answer the questions given below:

For the first time, the Indian Navy is participating in joint drills with the European Union Naval Force,
comprising warships from the French, Spanish and Italian navies, being held in the Gulf of Aden.

“A total of five warships from four navies are participating in the exercise on 18 and 19 June 2021. Other
warships include Italian Navy Ship ITS Carabiniere, Spanish Navy Ship ESPS Navarra, and two French
Navy Ships, FS Tonnerre and FS Surcouf,” the Indian Navy said in an official statement.

The two-day exercise will see high tempo-naval operations at sea, including advanced air defence and
anti-submarine exercises, cross-deck helicopter operations, tactical manoeuvres, boarding operations,
underway replenishment, search and rescue, man overboard drills, and other maritime security
operations.

"Ships of the four navies will endeavour to enhance and hone their war-fighting skills and their ability as an
integrated force to promote peace, security and stability in the maritime domain. Concurrently, a virtual
“information sharing exercise” is also being conducted between the Indian Navy Information Fusion Centre
Indian Ocean Region and Maritime Security Centre-Horn of Africa on June 18," the statement said.

Source: Article taken from India Today Written by: Abhishek Bhalla, Dated: 21 June, 2021

Which Naval exercise was recently conducted in the Gulf of Aden where the naval Forces of India, Italy,
Spain and France participated for the first time ?
A. Exercise EKUVERIN B. Exercise IN-EUNAVFOR
C. Exercise IN-EUKF D. Exercise Aden Ring of Fire
47. Where did India's first Information Fusion Centre (IFC) for the Indian Ocean Region (IOR) was
successfully inaugurated by the Indian Navy ?
www.byjusexamprep.com

A. Rohtak, Haryana B. Roorkee, Uttarakhand


C. Gurugram, Haryana D. Haldia, West Bengal
48. The Indian Navy and European Union Naval Force are a part of annual SHADE (Shared Awareness and
Deconfliction) meetings held annually at __________.
A. Bahrain B. Italy
C. France D. Spain
49. Approximately, what percentage of the world's total seaborne petroleum passes through the Gulf of Aden
on its way to the Suez Canal or to regional refineries ?
A. 17% B. 11%
C. 20% D. 5%
50. The Indian Navy and European Union Naval Force are also involved in naval assistance and deployments
under the charter of World Food Programme (UN WFP). Which of the following is/are correct regarding the
United Nations World Food Programme (UN WFP) ?
A. It works closely with the prominent Rome-based
UN agencies like the Food and Agriculture B. It's objective is to end hunger by protecting
Organization (FAO) & International Fund for access to food.
Agricultural Development (IFAD).
C. The program is helping in improving nutrition and
D. All of the above.
achieving food security goals across the globe.
51. Direction: Read the given passage carefully and answer the questions given below:

World Population Day is observed annually on [A] every year to highlight the problems of overpopulation
and raise awareness about the effects of overpopulation on the environment and development. There has
been a rapid increase in the world population over the last few decades and continuous population growth
can give rise to many other problems. Thus, to increase people's awareness of various population issues
such as the importance of family planning, gender equality, poverty, maternal health, and human rights,
World population day is observed every year.

Every year, on this day, various events and activities are organized, mostly in schools and colleges like
seminar discussions, educational information sessions, and essay competitions to draw attention to the
issues of the growing population.
www.byjusexamprep.com

Once upon a time, A.P.J. Abdul Kalam said that: "Almost half of the population of the world lives in rural
regions and mostly in a state of poverty. Such inequalities in human development have been one of the
primary reasons for unrest and, in some parts of the world, even violence".

Source: Article taken from India Today Dated: 10 July, 2021

World Population Day is observed every year on _______________ (redacted as 'A' in the passage).
A. 10 July B. 11 July
C. 07 June D. 29 June
52. What is the theme for World Population Day 2021 ?
B. "Overpopulation: First step towards
A. "Overpopulation : A serious concern"
compromisation of quality of life"
D. "Rights and Choices are the Answer: Whether
C. "Rights and Choices are the Problem: Strengthen baby boom or bust, the solution to shifting fertility
the Right to Reproductive Choices" rates lies in prioritising all people’s reproductive
health and rights."
53. In which year did the United Nations Population Fund (UNPF) be established as a trust fund ?
A. 1969 B. 1967
C. 1960 D. 1975
54. What is the title of the recently launched United Nations Population Fund’s (UNFPA) flagship State of
World Population Report 2021 ?
A. 'My Reproductive Choice is my Fundamental
B. 'I care my Body, I care my Reproductive Health'
Right'
C. 'My Body is My Own' D. 'I Own my Body'
55. The 'Uttar Pradesh Population Policy 2021-2030' was recently launched to reduce infant and maternal
deaths in a time-bound manner. Which of the following is/are the aim of the policy ?
A. To decrease the total fertility rate from 2.7 to 2.1 B. To increase modern contraceptive prevalence rate
by 2026 and 1.7 by 2030. from 31.7% to 45% by 2026 and 52% by 2030.
C. To decrease maternal mortality rate from 197 to
D. All of the above.
150 by 2026 and 98 by 2030.
56. Direction: Read the given passage carefully and answer the questions given below:
www.byjusexamprep.com

The cybersecurity firm NSO Group on Thursday confirmed the inspection of its offices by Israeli authorities
and said the company was “working in full transparency” with the representatives from Israel probing into
the Pegasus snooping row.

A team from the Israeli Ministry of Defence visited the NSO Group headquarters on Tuesday to conduct
an inspection as part of its probe into the Pegasus snooping row. NSO Group sells the Pegasus spyware.
Israel earlier appointed an inter-ministerial team to assess media reports following an investigation by 17
media organisations, which said NSO's Pegasus software had been used in attempted and successful
hacks of smartphones belonging to journalists, government officials and human rights activists.

While Israel has said it is looking into the issue with the utmost seriousness, NSO has maintained that the
report about Pegasus was “full of wrong assumptions and uncorroborated theories”. Pegasus is intended
for use only by government intelligence and law-enforcement agencies to fight terrorism and crime, the
company has said.

Source: Article taken from: indiatoday.in, Written by: Gita Mohan, Dated: 29 July, 2021

In which year did the Israeli firm 'NSO Group' established who developed the Pegasus spyware ?
A. 2010 B. 2005
C. 2012 D. 2016
57. Where was the '12th India Security Summit' held in 2019 with the theme "Towards New National Cyber
Security Strategy" (latest edition till date) ?
A. Bengaluru B. New Delhi
C. Mumbai D. Vadodara
58. Who is the current Secretary of the Ministry of Electronics and Information Technology (MeitY) ?
A. Devusinh Jesingbhai Chauhan B. Ajay Prakash Sawhney
C. Surinderjeet Singh Ahluwalia D. V.K. Bishnoi
59. Which of the following is an international treaty that seeks to address Internet and computer crime
(cybercrime) by harmonizing national laws ?
A. St. Lucia Convention on Cybercrime B. Budapest Convention on Cybercrime
C. International Convention on Cybercrime and
D. Paris Convention on Cybercrime
Cybersecurity
60. Where is the headquarters of the International Telecommunication Union (ITU) which is a specialized
www.byjusexamprep.com

agency of the United Nations for information and communication technologies ?


A. Geneva, Switzerland B. Vienna, Austria
C. Vladivostok, Russia D. Jerusalem, Israel
61. Direction: Read the given passage carefully and answer the questions given below:

This year’s The State of Food Security and Nutrition in the World (SOFI) summarizes the first global
assessment of food insecurity and malnutrition for 2020 and offers some indication of what hunger and
malnutrition would look like by 2030, in a scenario further complicated by the enduring effects of the
pandemic.

Nearly one-tenth of the world population – up to 811 million people – went hungry in 2020. After remaining
virtually unchanged for five years, world hunger increased last year. Further, it is projected that around 660
million people may still face hunger in 2030, 30 million more people than in a scenario in which the
pandemic had not occurred, due to lasting effects of COVID-19 on global food security. The setback
makes the achievement of the Sustainable Development Goal for zero hunger and ending all forms of
malnutrition more challenging.

The report indicates that progress has been made for some forms of malnutrition, but the world is not on
track to achieve any global nutrition targets by 2030.

Source: Article taken from: reliefweb.int, Dated: 21 July, 2021

Which of the following bodies has recently released the 'Global Food Policy Report 2021'?
A. World Health Organisation (WHO) B. Food and Agriculture Organization (FAO)
C. International Food Policy Research Institute D. International Fund for Agricultural Development
(IFPRI) (IFAD)
62. Which of the following are the benefits emanating from One Nation One Ration Card scheme (ONORC)
launched by the Government of India ?
B. Empowering consumers by giving the opportunity
A. Interoperability of Ration Card
to opt for the dealer of their choice.
C. Reducing social discrimination; particularly for
D. All of the above.
women and other disadvantaged groups.
63. Which rank did India secure among 107 countries of the world in the 'Global Hunger Index 2020' under the
‘serious’ hunger category with a score of 27.2 ?
www.byjusexamprep.com

A. 72nd B. 68th
C. 94th D. 57th
64. What is the purpose of the recently launched 'Intensified Mission Indradhanush (IMI) 3.0 Scheme' by the
Government of India ?
A. To provide free health-care benefits to the
B. To provide free affordable maternity care in private
children who lost their earning members of the family
hospitals.
during COVID-19 Pandemic.
C. To cover children and pregnant women who
missed routine immunisation during the Covid-19 D. None of the above.
pandemic.
65. Who is the current Chief Executive Officer (CEO) of the Food Safety and Standards Authority of India
(FSSAI) ?
A. Rita Teoatia B. Arun Singhal
C. V.K. Mahajan D. Sudhir Bhargav
66. Direction: Read the given passage carefully and answer the questions that follow:

The United Nations defines violence against women as "any act of gender-based violence that results in,
or is likely to result in, physical, sexual, or mental harm or suffering to women, including threats of such
acts, coercion or arbitrary deprivation of liberty, whether occurring in public or in private life."

According to Section 2(f) of the Domestic violence Act 2005, "domestic relationship" means a relationship
between two persons living in a shared household. Domestic relationships can be through marriage such
as wives, daughter-in-law, sisters-in-law, widows and any other members of the family; or blood
relationships such as mothers, sisters or daughters; and other domestic relationships including through
adoption, live-in relationships, and women in bigamous relationships or victims of legally invalid marriages.
Expression "relationship in the nature of marriage" cannot be construed in the abstract. It is to be taken in
the context in which it appears and to be applied bearing in mind the purpose and object of the DV Act as
well as the meaning of the expression "in the nature of marriage", (Indra Sarma v. V.K.V Sarma, (2013)).

Shared household- According to Section 2(s)of DV Act 2005, a shared household is where the aggrieved
person or a woman lives in a domestic relationship, either singly, or along with the man against whom the
complaint is filed. It may also imply a household where a woman has lived in a domestic relationship but
has been thrown out. This may include all kinds of situations whether the household is owned by the
www.byjusexamprep.com

respondent or it is rented accommodation. It also includes a house either owned jointly by the aggrieved
person and the respondent or both may have jointly or singly, any rights, titles or interests. The DV Act
recognizes a woman's right to reside in a shared household. This means a woman cannot be thrown out
of such a household except through the procedure established by the law. In case she is thrown out she
can be brought back again after obtaining the order from the court. A woman to claim the protection of
right in a "shared household" has to establish: (a) that the relationship with the opposite party is a
"domestic relationship", and; (b) that the house in respect of which she seeks to enforce the right is
"shared household".

The definition of Domestic Violence (DV) has been explicitly provided in Section 3 of the DV Act includes
the following acts of domestic violence. The Section also defines the meaning of terms physical abuse,
sexual abuse, verbal and emotional abuse, and economic abuse. It further enunciates that the overall
facts and circumstances of the case shall be taken into consideration in order to determine whether any
act, omission, commission or conduct of the respondent constitutes "domestic violence" under the said
section.

Source: Extracted, edited and recreated from: scconline.com, casemine.com, indiankanoon.org.

X used to live along with her husband Y, in her matrimonial home. X was being harassed by her husband
and her in-laws. One day, X was thrown out of the house alleging her of committing adultery. During this
period, Y moved out of the house (which was registered under Y's parents' name) and settled at a new
place. X brought an action against Y for the right to a shared household to the old home where Y used to
stay. Y contended that X no longer stayed and that was not the shared household. Decide the legality in
this case.
A. X can successfully seek the 'right to shared
household' because her relationship with Y falls
B. X cannot seek 'right to shared household' as X no
within the ambit of "domestic relationship" and that
longer stayed there and had no right in that house.
the house in respect of which she seeks to enforce
the right is "shared household".
C. X cannot seek 'right to shared household' as she D. X can successfully seek 'right to shared
committed adultery and deserved the right household' as she was thrown out of the house
www.byjusexamprep.com

punishment. without her consent and she has every right to live
there.
67. X used to stay in her husband Y's house. Y had an adopted sister named Z. Z used to harass and beat X.
X filed a case for domestic violence. Does the relationship of X and Z come under domestic relationship ?
A. No, as Z is an adopted sister and not an inherent B. None of the others are included because the
member of the family, she doesn't come under Domestic relationship is the relationship between the
domestic relationship. husband, wife and the husband's parents.
C. Yes, the relationship comes under domestic D. Can't say clearly as it depends on the property
relationship. rights shared between Y and Z.
68. 'P' and 'Q', two bachelors, were in a living relationship for 5 years. One fine morning, Q left a letter asking
P to empty the place immediately saying he was no longer interested in the relationship and paid back her
share of rent. As P failed to comply, Q forcefully removed her from the house. Q filed a complaint under
the domestic violence act for the right to a shared household. Decide accordingly.
A. P cannot enforce the right to a shared household B. P can enforce the right to a shared household
as both are bachelors and not married. irrespective of their marital status.
D. P can return back to her parent's place or stay at
C. P cannot file a complaint under domestic violence
a different place because living relationships end in
act but can approach the court under IPC 1860
vain.
69. According to the theme of the passage, select the wrong statement regarding Domestic Violence Act,
2005 from the following legal scenarios :
A. Domestic violence includes physical and sexual B. Section 3 of domestic violence act 2005 defines
violence but not mental violence. physical abuse.
D. According to Section 2(s)of DV Act 2005, a
C. According to Section 2(f) of Domestic violence
shared household is where the aggrieved person or
Act 2005, "domestic relationship" means a
a woman lives in a domestic relationship, either
relationship between two persons living in a shared
singly, or along with the man against whom the
household.
complaint is filed.
70. Which of the following statements regarding shared household is false ?
A. A woman to claim the protection of rights in a B. A woman to claim the protection of rights in a
"shared household" has to establish that the "shared household" has to establish that the house
relationship with the opposite party is a "domestic in respect of which she seeks to enforce the right is
relationship". a "shared household".
www.byjusexamprep.com

C. A woman cannot be thrown out of such a D. Shared household law exists only for the initial 20
household except through the procedure established years of the marriage.
by the law.
71. Direction: Read the given passage carefully and answer the questions that follow:

On September 27, 2020, the president of India Mr. Ram Nath Kovind gave his assent to the three farm
reform bills which are touted to play a major role in revolutionising the development of farmers in India.
Before understanding the bills, a brief look into the past introductions is necessary. APMC (Agriculture
Produce Market Committee) Act was introduced in 1960's at the very same time when the green
revolution started in India. APMCs set up Mandis or Markets across India where farmer's produce was
sold. Government gives licenses to these Middlemen; shops, storage facilities etc. are provided to them in
APMC markets. Many people work in these APMCs, there is storage of grains, so it requires laborers,
accountants so overall it is a self-thriving ecosystem. One thing which should be noted here is these
APMC markets are regulated by state governments, a tax is charged on each transaction so in a way the
government knows at what price produce is being sold.

Voice arose from time to time to remove these 'middleman' defects and in response the government
brought the three Acts in 2020. These three farm Acts seek to replace ordinances issued in June 2020.

1) The Farmers Produce Trade and Commerce (Promotion and Facilitation) Bill, 2020 (FPTC)- The FTPC
seeks to break the monopoly of government-regulated mandis, reduce reliance on APMCs
(disintermediation or reducing the influence of 'middlemen' or Angadias) and promote 'One India, One
Agriculture Market'. Intrastate and Interstate trade are now facilitated freely through this and governments
cannot levy barriers like market fee outside APMC (Agriculture Produce Market Committee) areas.

2) The Farmers (Empowerment and Protection) Agreement of Price Assurance and Farm Services Bill,
2020 (FAPAFS)- The FAPAFS bill provides a legal framework for farmers to enter into written contracts
with companies and produce for them. Combined with the first bill, it leaves the market open for private
players to join. As per its Preamble, it allows "national framework on farming agreements that empowers
farmers to engage with agri-business firms, processors, wholesalers, exporters or large retailers for farm
services and sale of future farming produce at a mutually agreed 'remunerative price'.

3) The Essential Commodities (Amendment) Bill, 2020- It takes away cereals, pulses, oilseeds, edible oils,
onion and potatoes from the list of essential commodities. Therefore, these commodities are now free of
the Essential Commodities Act restrictions and stand deregulated. However, the central government has
www.byjusexamprep.com

retained the right to regulate them under extraordinary circumstances, such as in case of a war, famine,
natural calamity, and impose stock limits if there is a steep rise in prices. It enables hoarding and free
trade now.

Source: Extracted, edited and recreated from: livelaw.in, theindianexpress.com.

Hari, a farmer, cultivated potatoes. At the end of a good potato season, a number of private companies
approached Hari to buy his potatoes at various rates. Never had Hari sold his potatoes anywhere apart
from the state run APMC mandis which provided a minimum selling price (MSP). Hari was informed by a
friend, now that the Farm bill 2020 was passed MSP and government procurement no longer existed.
Which of the following scenarios will help Hari ?
A. With the introduction of the private sector to buy B. MSP and government procurement will continue
products from farmers, MSP and government the same way as before, and Hari can sell his
procurement no longer existed. tomatoes in local APMC mandi.
C. Hari can sell his potatoes in APMC mandis, but D. Hari has the choice of selling his potatoes to the
MSP no longer exists and the price is decided by highest bidder. Therefore, approaching mandi for
average demand rate. MSP is useless.
72. Raghav, a farmer from Haryana, used to sell his products in APMC mandi. The price of tomato in the
mandi was Rs. 6/kg. Raghav was utterly disappointed. What other possible ways can Raghav seek ?
(Assuming that Farm Bills 2020 are passed and implemented).

I. Raghav can seek to sell his products to any private buyer outside the mandi without paying market fee.

II. Raghav can seek to sell his goods across the border of his state/district without any special charges.

III. Raghav can bargain and ask the mandi to increase the price. If not satisfied Raghav can file a
complaint in farmer dispute redressal forum.

IV. Raghav can seek to sell his products outside the mandi, but should obtain a special permission from
the concerned government authorities.

Mark your answer as:


www.byjusexamprep.com

A. Only I B. Only I and IV


C. Only I, II and III D. Only I and II
73. According to the comprehension mentioned above, which aspect of MSP has changed from 1960 to
present day ?
B. Middlemen started exploiting farmers, they formed
A. The effectiveness of MSP has reached a plateau cartels or an understanding among themselves and
during the last 20 years. started buying the produce at MSP only and sold to
traders at a high rate.
C. Middlemen started exploiting the farmers, they D. MSP has improved the state of farmers to great
formed cartels or an understanding among extent and has brought health, wealth and prosperity
themselves and started charging extra taxes secretly in recent years. Thus, farmers in Haryana and
above the MSP. Rajasthan are protesting.
74. Every year, monsoon rains across India lead to flooding in various states. This led to the death of potatoes
and the price of potatoes skyrocketed. Decide the role of central government in controlling the price and
situation. (Assumption is same as in the previous question)
B. The Central government under The Essential
A. The Central government should ask the state
Commodities (Amendment) Bill, 2020 has the right to
governments to set a bracket amount for the selling
decrease the prices of potatoes to the benefit of the
of potatoes.
customers.
C. The Central government under The Essential D. The Central government has no power under The
Commodities (Amendment) Bill, 2020 has the right Essential Commodities (Amendment) Bill, 2020 but
to stock limits if there is a steep rise in prices. state governments do.
75. According to the passage, which of the following are the correct set of reasons for the concern/
dissatisfaction of farmers with the introduction of Farm Bills, 2020 ?
A. The MSP, Introduction of corporate/private B. The MSP, Loss of revenue for the state, Arbitrary
entities, more power to central government, Loss of control by central government, Loss of revenue to
revenue for the state. the state, Loss of jobs for the middle man.
C. The MSP, Introduction of corporate/private D. Introduction of the corporate/private sector, Loss
entities, Loss of revenue to the state, loss of job for of jobs for middlemen, Burden on farmers, Arbitrary
middle man. control by the centre.
76. Direction: Read the given passage carefully and answer the questions that follow:
www.byjusexamprep.com

A contract is an agreement between two or more parties creating obligations that are enforceable or
otherwise recognizable at law. Execution of these obligations may be affected by unforeseen or
supervening events, i.e., events which are unexpected or incapable of being known in advance by either
of the parties and which ultimately discharge the parties from their contractual obligations. The doctrine of
frustration is a "doctrine" of a special case of the discharge of contract by an impossibility to perform it.
The Indian Contract Act, 1872 ("Contract Act) does not define the term frustration. The Black's Law
Dictionary defines frustration in relation to contracts as "the doctrine that if a party's principal purpose is
substantially frustrated by unanticipated changed circumstances, that party's duties are discharged and
the contract is considered terminated," also termed as the frustration of purpose. In India, courts nuanced:
"The expression 'frustration of the contract' is an elliptical expression. The fuller and more accurate
expression is 'frustration of the adventure or of the commercial or practical purpose of contract'". This
doctrine is a device to reconcile the rule of absolute contracts with a special exception which is demanded
in certain circumstances in the name of justice.

The doctrine comes within the purview of section 56 of the Contract Act as it discharges the contract by
reason of supervening impossibility or illegality of the act agreed to be done. A contract is also frustrated
under section 32 when the condition, on which the contract is contingent, is not fulfilled or cannot be
fulfilled because of impossibility (paragraph 1 and 2 of section 32, respectively). Nevertheless, the doctrine
under Indian law is associated with section 56. As section 32 only applies when contracts are discharged
and parties absolved of their obligations as per terms already contained in the relevant contract. Section
56 applies when contracts are discharged and parties absolved of their obligations as a result of
subsequent impossibility due to outside forces and factors. Section 56 states that an agreement to do an
act which becomes impossible or unlawful is void. The word "impossible" has not been used in the sense
of physical or literal impossibility. The performance of an act may not be literally impossible but it may be
impracticable and useless from the point of view of the object and purpose which the parties had in view.
Therefore, if an untoward event or change of circumstances totally upsets the very foundation upon which
the parties rested their bargain, it can very well be said that the promisor finds it impossible to do the act
which he promised to do.

Source: Extracted, edited and recreated from: www.casemine.com, lawcutor.com, livelaw.com.


www.byjusexamprep.com

Krishna Pratap, the defendant, was keen on watching the coronation ceremony of the king. Therefore,
Krishna rented a flat for three days as he could watch the ceremony from the balcony of the flat. He
informed Aniket about the reason for renting the flat. But due to ill health of the king, the ceremony was
cancelled. Krishna refused to pay the rent. Decide the liability/non-liability of Krishna.
B. Krishna is not liable to pay the rent as the doctrine
A. Krishna is liable to pay the rent irrespective of the
of frustration applies, the object of contract as
coronation as the object of the contract was to reside
recognised by both the parties was to witness the
and residence was possible.
coronation ceremony and it was impossible.
C. Krishna is liable to pay the rent and the doctrine D. Krishna is not liable to pay the rent as he did not
of frustration does not apply as the plaintiff rented reside in the flat, but doctrine of impossibility does
the flat with the view of residence. not apply.
77. X entered into a contract with Y to supply 200 litres of petrol. Due to some mishappening in the middle
east, the price of petrol rose 300%. X could not supply petrol. Y sued X for specific performance. X
pleaded doctrine of impossibility. Decide the consequence thereafter.
A. X can plead doctrine of impossibility as the price B. X cannot plead doctrine of impossibility as rise
escalated tremendously high and subsequently and fall of prices is normal and expected in the
making it impossible for the defendant to supply. market.
D. X can plead to the court to delay the date of
C. X can supply petrol once the prices come down
supply but cannot withdraw himself from the
and there is no need to go to court.
contract.
78. Harish joined as a manager for ten years, and undertook a vote not to enter into any professional
engagement without the consent of the employer. Before the completion of the said term, he was asked to
join military service. After the war, Harish undertook professional engagement and was sued by his
employer. Harish pleaded for frustration of contract. Decide accordingly.
A. Harish cannot plead frustration as until the
B. Harish cannot plead frustration as enrolling in the
completion of 10 years he was under the contract
military is no ground for frustration of contract.
and was supposed to abide by it.
C. Harish can plead frustration as from the time he
joined the military he was no longer a manager and D. All contracts stand rejected when one joins the
was not bound to the contract. Thus, the contract military as it is for the service of the nation.
was frustrated.
79. According to the crux of the passage, why does the doctrine of frustration come only under section 56 and
www.byjusexamprep.com

not under section 32 of Indian contract act 1872 ?


A. Section 56 gives specific explanation about
B. Section 32 applies only when the contracts are
impossibility while section 32 discusses only one
not discharged.
particular condition.
C. Section 32 only applies when contracts are
discharged and parties absolved of their obligations D. Doctrine of frustration applies both under section
as per terms already contained in the relevant 32 and 56. The above given statement is incorrect.
contract.
80. As per the legality of the passage, which of the following is an impossible event under section 56 of the
Indian Contract Act 1872 ?
B. Shivam orders a brand -new car which was to be
A. Sam invites Tom to play guitar at his ceremony. imported from France. On the day of export from
Tom has a bad fight with his girlfriend on the day of France, the flights get cancelled due to
the ceremony and fails to attend the programme. thunderstorms. The car company cancels the order
While sued, Tom pleads doctrine of impossibility. and pleads doctrine of impossibility in court while
sued.
C. Suraj, signs a contract with singer Zehan Malik to
perform at his place. On the day of the programme, D. Akbar failed to attend all import projects as his car
Zehan suffers a mild cardiac attack and is admitted broke down. While sued, he pleads frustration under
to the hospital. Suraj sues Zehan, Zehan pleads section 56.
impossibility.
81. Direction: Read the given passage carefully and answer the questions that follow:

In common law, assault is a tort, an act of the defendant which causes to the plaintiff reasonable
apprehension of the infliction of a battery on him by the defendant. When the defendant creates his act by
an apprehension in the mind of the plaintiff that he is going to commit battery against the plaintiff, the
wrong of assault is completed. The wrong consists of an attempt to do harm rather than the harm being
caused thereby. In assault, charges must include conduct that is offensive which is offensive or causes
another person to the fear of their safety. This clearly means that one can be guilty of assault even if
he/she did not physically harm the victim. In the case of R. v. S George, it was held that the pointing of a
loaded gun to another is an assault. If the pistol is not loaded, then even it may be an assault, if pointed at
such a distance that it may cause injury. if a person advances the manner of threatening to use force, then
there is assault. This was decided in the case of Stephens v. Myers.
www.byjusexamprep.com

If one or more elements have not been satisfied then It can be a defence to an assault charge. Elements
of the crime of assault are: (I) An act or conduct intended to create: To prove a criminal attack, the
defendants' behaviour must be motivated to create a situation of fear or danger in the victim's mind.
Accident acts do not include allegations of assault. (II) A reasonable apprehension: Further, the victim
must reasonably believe that the defendant's conduct will harm or humiliate him. The victim must
understand the defendant's potentially harmful or offensive acts. (III) Of imminent harm: The victim's fear
must be a direct response to a threat that is imminent. Future threats, such as "I will beat you tomorrow,
will not result in assault charges. In addition, there must be some kind of perceived physical threat to the
victim in the loss; For this reason, words by themselves generally do not constitute an attack.

It is believed that the defendant's actions would cause physical danger or abusive behaviour to the victim.
Thus, the pretence of kicking or punching the victim may be an attack, as will attempt to spit on the victim
(aggressive behaviour). All of the above elements must be present and the evidence must be supported
with evidence if found guilty for the attack. It can be difficult to prove whether the defendant actually
intended the attack. Similarly, judges often spend a lot of time determining whether a defendant's actions
are considered harmful or abusive. In determining this, they will consider what an average person may
perceive as harmful or aggressive.

Source: Extracted, edited and recreated from: legalserviceindia.com, mondaq.com, scconline.com.

Sudhir was sitting in his backyard. Bhupesh who was inside a passing train shoved his fists towards
Sudhir citing he will bash him. Did Bhupesh commit assault on Sudhir ?
B. Bhupesh did not commit assault as there was no
A. Bhupesh did not commit assault as he was inside
reasonable apprehension of attack nor an imminent
the passing train.
threat.
C. Bhupesh committed assault on Sudhir D. Bhupesh committed assault as he committed an
irrespective of whether he was inside the train or not. act intending to cause hurt to Sudhir.
82. Ritik was walking in a street of his locality. Karan starts running at Ritik with an axe in his hand. Ritik
escaped and filed a suit for assault. Decide accordingly.
A. Karan will be liable for assault as all the B. Karan will not be liable for assault as there was no
requirements for assault are fulfilled. imminent harm present.
D. This case cannot be tried under common law/law
C. Karan will not be liable for assault but for battery.
of torts.
83. X and Y were neighbours. They shared a common boundary. One day they had a dispute regarding the
www.byjusexamprep.com

land border. The matter accelerated and X said he would bring a knife to kill Y and started walking towards
his house. Fearing of assault, Y hit X with a stone and ran away. Whether the fear of assault was present
in this case or not ?
A. The fear of assault was present as X was going to B. The fear of assault was present as X was about to
bring a knife. Y's act was to defend himself. commit a battery at Y.
D. Y instead of hitting with a stone should have
C. The fear of assault was absent as there was no approached a police station and filed a complaint,
fear of imminent harm. which is a more objective way to deal with such
circumstances.
84. What nature of the tort 'Assault' can be deduced from the case of R v. S George mentioned in the passage
above ?
B. To charge a person with assault it is not
A. One can be charged with assault only when his
necessary that his act may possibly lead to hurting
act is capable of committing hurt to the other person.
others. Reasonable apprehension is enough.
C. To charge a person with assault he or she should
D. Person who commits battery will necessarily also
have the intention to hurt. Without intention it is not
commit assault. Assault is a part of the battery.
assault.
85. Which of the following inferences is/are correct about the elements of the crime of assault as per the
principles of the passage ?
A. While proving a criminal attack, the defendants' B. In the case of assault, the victim must reasonably
behaviour must be motivated to create a situation of believe that the defendant's conduct will harm or
fear or danger in the victim's mind. humiliate him.
C. The victim's fear must be a direct response to a
D. All of the above.
threat that is imminent.
86. Direction: Read the given passage carefully and answer the questions that follow:

Section 375 of IPC defines the offence of rape with the help of six descriptions. One of the exceptions to
this offence is “Sexual intercourse or sexual acts by a man with his own wife, the wife not being under 15
years of age, is not rape”.

Marital Rape refers to unwanted intercourse by a man with his wife obtained by force, threat of force, or
physical violence, or when she is unable to give consent. Marital rape occurs irrespective of age, social
www.byjusexamprep.com

standing, education, race or ethnic origin. India is one of the few countries in the world continues to
exempt husbands from being charged with rape committed against their wives.

One of the basic causes of marital rape is that in a male dominant relationship many husbands use sexual
violence as a way to maintain power. Another important factor for raping an unwilling pair-bonded mate
may be a ‘male tactic of sperm competition’ which is a result of sexual jealousy of a man as a woman's
sexual unreceptivity may suggest to him that she is having consensual intercourse with another. Also
abusive men with the history of alcohol or drug problems are apt to abuse their spouses sexually when
drunk or sober.

Many factors contribute to the difficulty that many women have in recovering from marital rape. First is the
fact that the rapist is a close acquaintance and most of the women are unemployed and dependent upon
their husbands for their living and thus are unable to resist such a dreadful act. The survivor may not
recognize that she was sexually violated, and so may think that her feelings of fear or anger mean that
she is crazy or somehow flawed. Because the perpetrator was at one time trusted and loved, the survivor
is likely to deal with bitter feelings of betrayal and broken trust. Feelings of grief and loss are common for
victim of marital rape because the rape is a break in an important relationship. Lack of support from family
and friends is also often a problem. Some other difficulties may arise for a woman if the assailant/husband
is also the economic provider for the family, or the father of her children. For some women, the ability to
"keep a man" is important to feelings of self worth. All these entanglements and more, along with the
religious and social beliefs, may function to keep women in marriages where they are raped.

Having considered the inadequacies of the IPC and the half-hearted efforts of Law Commissions, the
conclusion is inescapable that 'male' legislators are still not willing to shed their notion of marriage being
the legitimating of all sexual intercourse with women, for reasons of upholding patriarchy and denying
sexual agency to women. Such backwardness of rape laws in India gets further enhanced, when seen in
stark contrast to the rape laws in most other jurisdictions where marital rape has already been
criminalized.

Source: Extracted, edited and recreated from: indianexpress.com, legalserviceindia.com.


www.byjusexamprep.com

As per the author's viewpoint and legality of the passage, “male tactic of sperm competition” results from
which of the following sources ?
A. The fact that it is through sexual violence that
B. The natural dominant nature of the male sex.
they can establish power.
C. The fact that it is the tendency of the male sex to D. The lack of a positive response of a woman to sex
spread their sperm for offsprings. may suggest to him of infidelity causing jealousy.
87. X is a newly married wife who has gone with her husband after marriage to Bangkok on a trip. On a
number of occasions X had refused sexual intercourse with her husband due to her not being mentally
prepared for the same. On a certain evening in Somana, while they are at a party, her husband, Y gets her
drunk on a lot of wine, and thereafter takes her to her hotel room and then forces her to have sexual
intercourse with him and then finally rapes her with her not being able to respond. Decide the
consequence thereafter.
A. Y has not done anything wrong since they are a
couple and it is natural for them to engage in sexual B. Y has commited marital rape on his wife.
intercourse.
C. X being Y's wife is under a duty to sexually please D. X had not refused to engage in a sexual act, since
her husband. she did not forbid him also.
88. Following the facts of the previous question, and the information given in the passage, what would be the
scenario if the husband Y had threatened his wife X that he would desert her, if she did not engage in
sexual intercourse with him, and she complied ?
A. Y would not be guilty of marital rape as he did not
B. Y would be guilty of threatening to abandon his
use physical violence on his wife to have sexual
wife.
intercourse with him.
D. Y would not be guilty of marital rape as his wife
C. Y would not be held guilty of marital rape. ought to have sexual intercourse with him given the
fact that she was married to him.
89. Which among the following is/are the most appropriate reason(s) for the woman raped not to realise that
she has been sexually violated and has to deal with betrayal and broken trust ?
B. Because of the lack of support from family and
A. Because the perpetrator may be a loved one.
friends.
D. All of the above are appropriate reasons for this
C. Because the wife is unemployed.
cause.
www.byjusexamprep.com

90. As per the author's point of view in the passage, what is the most fundamental reason as to why there is
no remedy for marital rape in India ?
A. Inadequacies of the law and the attitude of B. The ability to “keep a man” is linked to a woman’s
legislators. self worth so marital rape goes unpunished.
C. Patriarchal principles and the dependence on D. Lack of international framework to deal with the
husbands. issues related to marital rape.
91. Direction: Read the given passage carefully and answer the questions that follow:

In Netherlands, euthanasia is sanctioned by the passage of “Termination of Life on Request and Assisted
Suicide (Review Procedures) Act” 2002 providing well defined guidelines for the same. Belgium was the
second nation to take a stand in this direction. In Switzerland assisted suicide can be performed by
nonphysicians . Both Active Euthanasia and Assisted Suicide are legal in Luxembourg since 19th
February 2008. Apart from these handfuls of nations, none in Europe have legalized Active Euthanasia or
Assisted Suicide.

The issue of euthanasia, or assisted dying, is incredibly controversial and there are legitimate concerns on
either side of the debate. The starting point has to be in the law, which at present is failing, as shown by
the recurrence of cases in the courts that often place relatives, already dealing with the painful loss of a
loved-one, in the middle of distressing legal battles. There is clearly a desire; whether we like it or not ;
among a number of patients at the end of often terrible battles with debilitating, incurable diseases to end
their suffering with the support of their relatives. To deny this right is to prolong the suffering for individuals
and families, something that I can simply not condone.

I do accept though that this is not like any other clinical decision and that if society is to offer this solemn
choice it must also built in safeguards to its laws that not only rectify the inadequacies of the current
situation, but also protect the vulnerable, the weak and all those doctors and nurses included who are
involved in this incredibly difficult situation.

As a start we must enact legislation to decriminalise acts of euthanasia and physician-assisted suicide. To
allow a terminally ill individual to end their life is the only humane, rational and compassionate choice. The
terminally ill are travelling abroad to countries where the right to end of life in terminal cases is recognised
and is lawful. We must not prosecute loved ones for "encouraging or assisting" suicide who enable or
assist a terminally ill individual to travel abroad to end his or her life lawfully.
www.byjusexamprep.com

Many people are opposed to legislation that would allow "end of life" choices. But our concerns relating to
abuses and protection of the vulnerable can be addressed by ensuring that certain objective safeguard
conditions are met prior to allowing a terminally ill individual from exercising his or her right to die with
dignity like The adult mentally competent patient must be terminally ill in severe pain and two independent
physicians must be satisfied that it is so.

The goals of the medical profession should continue to remain one of saving lives but this should not be at
the expense of compassion and a terminally ill individual's right to choose to end his or her life and die
with dignity.

Source: Extracted, edited and recreated from: latestlaws.com, legalserviceindia.com, livelaw.in.

Parallaly analysing the information given in the passage and the existing laws in India (regarding
euthanasia), which of the following is most appropriate inference ?
A. There is poor legislation in the country to deal B. There is legislation which deals with the physician
with the issue of euthanasia. assisted suicide.
C. There is no legislation to legalise the acts of
D. All of the above.
euthanasia and physician-assisted suicide.
92. What is the distress in the legal battles which the author refers to faced by the relatives of the patient ?
A. It is distressing since the law is not equipped to B. It is distressing since the relatives have to
deal with the issue and the relatives are already in undergo tedious litigation so as to allow euthanasia
great turmoil. to be performed.
C. It is distressing since these diseases are
D. All of the above.
incurable and debilitating.
93. Arjun's old father has been suffering from a neuromuscular disease which is a terminal illness. Seeing
such unfortunate condition, Priya, his childhood friend who is a doctor secretly, one day when Arjun's
father is in his clinic, administers a morphine injection so as to save the old man from such continuous and
terrible pain. Decide the liability of Priya, if any, in this case:
B. Priya has not performed a criminal act as he is a
A. Priya has performed an honourable act so as to
doctor and has administered the injection under
end the suffering of Arjun's father.
medical expertise.
C. Priya has committed medical negligence. D. Priya has performed a criminal act.
94. Following the facts of the previous question, if Arjun's father had given his consent by way of signing a
www.byjusexamprep.com

form that he is willing to end his life, then in such a case, what would the actions of Priya be considered ?
A. It is legal and valid as the outcome of the same
B. It is illegal since impairment did not leave Arjun's
was known to the victim, and his consent was
father to give his consent.
already taken.
C. It would still be a criminal act as consent is not an
D. None of the above.
exception.
95. Which of the following statements is the author most likely to agree with given the information in the
passage ?
A. We must always allow the practice of euthanasia B. We must allow people to travel abroad so as to
in all cases. undergo euthanasia or mercy killing.
C. We must build in checks and balances so as to
D. All of the above.
allow euthanasia conditionally.
96. Direction: Read the given passage carefully and answer the questions that follow:

As per Hindu Shastra, it is believed that the adopted son is a reflection of the natural son of Adoptive
Parents. This helps in guaranteeing the protection and care of the adopted son. Once a child for good is
separated from his biological parents and becomes a legitimate child of his adoptive parents, he has all
the rights that are related to adoptive parents. This means the adoptive child cannot marry the other
adoptive child or real child of his adoptive parents.

A Hindu adoption is not valid unless the man takes prior consent from his wife and there is a "ceremony of
giving and taking in adoption," the Supreme Court has held. A Bench led by Justice L. Nageswara Rao
said the mandate of the Hindu Adoptions and Maintenance Act of 1956 was that no adoption was valid
unless the two essential conditions of the consent of the wife and the actual ceremony of adoption were
established.

The March 6 judgment came on an appeal filed by M. Vanaja, who claimed she is the adopted daughter of
the late Narasimhulu Naidu. She had filed a civil suit for partition of property. The suit was dismissed. The
Hyderabad High Court had eventually upheld the dismissal, following which an appeal was filed in the
apex court. Justice Rao, who wrote the judgment, said Sections 7 and 11 of the 1956 Act are the consent
of the wife before a male Hindu adopts a child and proof of the ceremony of actual giving and taking in
adoption.
www.byjusexamprep.com

"The appellant [Vanaja] admitted in her evidence that she does not have the proof of the ceremony of
giving and taking of her in adoption. Admittedly, there is no pleading in the plaint regarding the adoption
being in accordance with the provisions of the Act. That apart, the respondent, who is the adoptive mother,
has categorically stated in her evidence that the appellant was never adopted though she was merely
brought up by her and her husband," the court concluded, rejecting the appeal.

Source: Extracted, edited and recreated from: blog.ipleaders.in, www.legalserviceindia.com,


www.thehindu.com.

X and Y are a childless couple. They adopted a child, Z from an orphanage nearby to their house. They
fulfilled all the legal requisites for it. They however, did not inform X's parents as they may not accept a
non-biological child in the family. A few years down the line X's parents get to know the truth. Decide
accordingly.
B. Z's adoption is valid and the fact of hiding her real
A. Z has to be given back to the orphanage from
identity from X's parents does not affect her legal
which she was adopted.
status.
C. Z will have to pay all the expenses borne on her D. X and Y should have tried artificial methods for
food, lodging and education after she turns a major. conceiving before adopting Z.
97. Satish was married to Shristi. During childbirth, Shristi died while the child survived. A year later the child
also died due to respiratory infection which leads to cardiac arrest. Later, Satish marries Suman. Due to
some hormonal problems, Suman is not able to conceive. Satish is upset and asks Suman permission to
marry someone else in order to have a child. Suman outrightly denies. Satish, without telling Suman,
adopts a male child, Sourabh. Decide the legality thereafter.
A. The adoption of Sourabh is valid under the law as B. The adoption of Sourabh is valid under the law as
Satish's first child had died. Satish's first wife had died.
D. The adoption of Sourabh is invalid under the law
C. The adoption of Sourabh is invalid under the law
as there are alternative therapies to correct
as Suman was not informed.
hormonal imbalances.
98. Harsh is married to Damini. Soon after marriage, Damini gives birth to a baby boy. They go to Himachal to
celebrate. While returning, their train meets with an accident. Harsh and Damini survive while their baby
dies. Damini suffers some injuries on her uterus which develops into a sepsis. The uterus had to be
www.byjusexamprep.com

removed in order to save her life. This saves Damini but takes away her ability to ever conceive in future.
Damini goes into depression. Harsh in order to console Damini suggests adoption. Damini was not willing
at first but later agreed. They adopt Kavya, a girl aged 6 years from an orphanage. All the formalities were
properly completed and Kavya was brought home. What is the most appropriate rationale in this case ?
A. Whether the adoption is legal or not depends B. Harsh and Damini should have consulted other
upon Kavya's decision after she turns a major. doctors for conception.
C. Kavya's adoption is not proper as Damini was not D. Kavya's adoption is proper and she is now a part
willing to adopt a child. of the family.
99. Kamlesh is married to Ragini. They do not have a child due to some problem with Kamlesh. One day
Kamlesh saw a small baby around 5 years crying for help near a temple. He asked him why he was
crying. The boy said his mother had refused to get him his favourite chocolate and went for puja inside the
temple. Kamlesh offered him the chocolate in return for which the boy had to go to Kamlesh's home. The
boy agreed. When Ragini got to know about this, she too approved of it. Henceforth, they treated the boy
as their adopted child and fulfilled all his desires. Decide the consequence thereafter.
B. The boy becomes the adopted child of Kamlesh
A. The boy becomes the adopted child of Kamlesh
and Ragini if he agrees to stay with them once he
and Ragini.
attains majority.
C. The boy is not an adopted child as the proper D. The boy is not an adopted child as he is only 5
procedure for adoption was not followed. years old.
100. As per the theme of the above mentioned passage, which of the following statements is/are incorrect
regarding the Adoption procedure under Hindu Law ?

I. To adopt a child under Hindu Law, the consent of the wife rarely matters but their consent is taken into
consideration for the sake of motherhood.

II. To adopt a child under Hindu Law, there must be a ceremony of giving and taking in adoption, as the
procedure established by law.

III. There must be an actual giving and taking of consideration and child should take place.

Mark your answer as:


A. Only I B. Only II
C. Only III D. Only I and III
101. Direction: Read the given passage carefully and answer the questions that follow:
www.byjusexamprep.com

The four essential elements of a crime are:- (1) the crime must be committed by a person, (2) there must
be hurt or injury caused to another, (3) there must exist an Actus Reus, (4) there must be a Mens rea to
commit the crime, with certain exceptions. But, the two major essential elements for criminal liability are -
'Mens Rea' and 'Actus Reus'. Mens Rea is the wrongful intention or some blameworthy condition of mind.
It is the conscious exercise of the mental faculties by a person to do an act, for the purpose of
accomplishing or satisfying a purpose. The importance of mens rea is brought out by the Latin maxim -
'Actus non facit reum nisi mens sit rea' which means that the act does not make one guilty, unless the
mind is also guilty.

Mens Rea is also implied in the sense that a man is presumed to intend the natural or probable
consequences of his own act. As a general rule Mens Rea and Actus Reus are sine qua non for attracting
criminal liability. Actus Reus is the actual wrongful act which causes harm, whether physical or mental, to
the victim or his property.

Some crimes require a deeper level of Mens Rea, like theft. Crimes like theft involve the specific intention
to deprive the rightful owner of the full use of the stolen goods and enjoyment of his personal property with
no intention of returning the said goods to their rightful owner. However, in cases of theft, the object does
not necessarily have to be removed from the victim’s immovable property (Example: – A person steals a
diamond ring belonging to ‘A’ and hides it in a flower vase, in ‘A’’s house with the intention of retrieving the
ring later and with no intention of returning it to ‘A’. The moment the ring is taken from the location where
‘A’ left it, theft has been committed, and the thief can be prosecuted.) Cyber crime is simpler since theft of
digital data cannot be accidental, and theft of identity, data and confidential information is done with
malicious intent to harm the reputation of the victim or to get wrongful gain from the use of someone else’s
property or resources.

Source: Extracted, edited and recreated from: blog.ipleaders.in, www.livelaw.in, www.lawcutor.com.

Mr. X kept 15 copies of a book by the name 'Odyssey Blue'. It was a popular book written by Marlon
Robert. The book contained obscene pictures and language. Selling of books containing obscene material
was a criminal offence. Mr. X was not aware about the contents of the book as there are hundreds of other
books in his store and therefore, as a book-keeper, he cannot be supposed to know the details of each
and every book. Mr. Y purchases one copy of the 'Odyssey Blue' from X's shop. What is the correct legal
consequence in this case ?
A. Mr. X does not have an actus reus. B. Mr. X does not have a mens rea.
www.byjusexamprep.com

C. Mr. X has both mens rea and actus reus. He is D. Mr. X has mens rea in this case.
therefore criminally liable.
102. Mr. X was travelling from Bangkok to Hongkong. He had a transit passage at Delhi Airport. He was
carrying a permissible quantity of gold slabs in his briefcase. He boarded his flight from Bangkok on 27th
November. On the same day the Government of India made carrying of gold slabs a punishable offence.
On his arrival to Delhi airport, the gold slabs were discovered and he is sought to be prosecuted. Decide
the legality in this case.
A. Mr. X does not have a mens rea. B. Mr. X has mens rea.
C. Mr. X is criminally liable as he has both the D. Mr. X cannot be prosecuted in India as he did not
elements i.e. actus rea and mens rea. board the flight from India.
103. Johnny loved a girl named Simran. Simran told Johnny her age to be 18 years but actually she was only
15 years of age. In the place where they lived it was a crime to take a minor girl, below 18 years, out of
her parent's custody without their consent. Johnny took Simran to a nearby Amusement Park without her
parent's consent. Decide accordingly.
A. Johnny has committed a crime as he took a minor B. Johnny has not committed a crime as he believed
girl out of her parent's custody without their consent. Simran to be 18 years old.
C. Johnny must apologize to Simran's parents and D. Simran has all the rights to go to any place and
seek their daughter's hand for marriage like an with anyone she likes as girls are not their parent's
honourable gentleman. property.
104. Sambhav was gifted a new car by his father on his securing first rank in National Entrance Eligibility Test.
Sambhav was fond of cars and he drove it out on a busy road. Sambhav, driving at a lightning speed,
jumps the red light hitting Shruti, who was crossing the road. Shruti was six months pregnant and was
coming back after shopping for clothes and other accessories for her soon to be baby. Shruti dies and so
does her baby. Decide accordingly.
A. Sambhav is not liable for the death of Shruti as it B. Sambhav is not liable for the death of Shruti as
is reasonable for a person to drive at top speed Sambhav did not know that Shruti was crossing the
when given a new car. road.
C. Sambhav is liable for the death of Shruti as she D. Sambhav is liable for the death of Shruti as his
was six months pregnant. act implied mens rea.
105. Which of the following is/are the correct inferences that can be rationally derived from the gist of the
passage ?
A. One of the elements of crime, i.e. mens rea is B. Actus Reus can also be the omission of an act, by
www.byjusexamprep.com

also implied in the sense that a man is presumed to omitting to do something that the accused knows he
intend the natural or probable consequences of his is bound by duty or law to do.
own act.
C. Some crimes do not necessarily require a
malafide intention towards one particular person,
D. All of the above.
and cases involving criminal negligence are a prime
example of this.
106. Direction: Read the passage given below and answer the questions that follow:

When Ariana Grande cried on stage recently, following her performance of an emotionally laden song, she
later took to Twitter to apologise and thanked her fans for accepting her humaneness. Producing
emotional tears is a uniquely human thing and yet, for many, our first reaction to crying is to apologise.

Public displays of crying and emotional release, especially of emotions deemed as unattractive like being
upset or angry, remain taboo. This is because there are socially accepted rules that govern the way we
feel things. These “feeling rules” guide the types of emotions and feelings deemed appropriate to display
at certain times and places. These rules tell us that is it acceptable to cry at funerals, but not necessarily
at pop concerts. Equally, such rules have often stereotyped certain cultures and genders into particular
norms. So feeling rules tend to dictate that men must show greater restraint in expressing their emotions
publicly.

The pressure of fast-paced, 24/7 societies has created a deficiency of times and places to release
emotion. And into this emotional void a marketplace has sprung up to provide people with places where
they can safely vent. Japan is at the forefront of this. The Japanese, often stereotyped as emotionless,
have found ways to cater to a growing demand for emotional release.

And it’s not just Japan that has an emotional release industry. Cities around the world have seen the
launch of anger rooms that provide a designated and safe space for customers to release rage through
destroying objects. The value of this kind of service space is evident, especially at a time when society
faces a mental health crisis, with men often worse affected by the inability to talk about or release their
emotions. Suicide is the number one cause of death for men under 50 in the UK and suicide rates among
US men is four times higher than women. So, it is important to create such spaces for men to vent out
their emotions, without worrying about the societal pressures.
www.byjusexamprep.com

What is the central idea of the passage as conveyed by the author?


A. Crying in front of large number of people is B. One must strictly adhere to the ‘feeling rules’ as
unattractive stereotyped by the society
C. The rise of emotional release industry and the
D. The emotional quotient of men is far different from
initiatives being taken to help release stress and
that of women
vent out emotions
107. As per the passage, which of the following approaches can be most effective to curb the issue of
increasing suicides in men?
A. The ‘feeling rules’ of the society must be
B. Public displays of crying and emotional release
reconsidered to allow freedom of expressing
must be made frequent
emotions irrespective of the gender
C. More of counselling sessions must be arranged D. More of emotional release spaces must be
for men to help them release their stress created for men who face mental health crisis
108. Which of the following notions is expressed in the passage to enunciate the significance of emotional
release spaces?
A. The value of such spaces becomes necessary at B. The fast-paced lifestyles have led to the rise in
a time when society faces a mental health crisis. stress levels.
C. Japan is catering well to the growing demand for D. Such spaces provide a designated and safe
emotional release. space for customers to release rage.
109. “These “feeling rules” guide the types of emotions and feelings deemed appropriate to display at certain
times and places.”

Which of the following conclusions can be drawn from the above statement?
A. Emotions are controllable. B. Time significantly decides the emotion
C. The ‘Feeling Rules’ offer a comprehensive guide D. Emotions make humans and embrace humanity
to all emotions and not vice versa
110. Which of the following weakens the author’s idea of “Producing emotional tears is a uniquely human thing
and yet, for many, our first reaction to crying is to apologise.”
B. Society looks down upon and does not honour
A. Only humans shed tears.
those who shed tears all the time
C. Apologizing is the best thing one should do, when D. One must not regret for being emotional in front of
one sees someone exhibiting emotions through anyone.
www.byjusexamprep.com

tears as it is not a part of good conduct.


111. Direction: Read the passage given below and answer the questions that follow:

The main task of education and research is to train people to perform in future economic and
technological environments with many unknowns. Workers prepare to tackle unknown problems using
instruments yet to be developed. In truth, we know embarrassingly little about tomorrow’s jobs.
Interdisciplinary and holistic education and research are indispensable as the workplace combines
cognitive skills with teamwork and debate with focus on adaptability, replacing silo thinking with a flexible
approach that applies knowledge from multiple sectors that at first appearances may not seem relevant.

Education and research are increasingly out of touch with demand for skills. The tendency to focus on
cognitive skills, including the STEM topics of science, technology, engineering, and mathematics, cannot
preclude the productivity benefit of soft skills – applying knowledge and finding opportunities offered by
technology. In recent decades most countries have fallen into the trap of overextending cost-benefit
analyses while training students to solve yesterday’s problems. Governments enforce short-term fiscal
planning on education programs, yet measuring social skills is not as easy as calculating STEM
competences.

Much attention is devoted to high-calibre education and research and for good reason, but demand trends
suggest that human factors may be more essential than normally assumed. For example, health, nonstop
improving of skills and entertainment may prove to be the growth sectors of the future – and main job
providers. Human maintenance will grow almost exponentially steered by demographics and a growing
proportion of elderly citizens. Human improvement, the ability to try new technologies should not be
overlooked as higher productivity embedded in new technology only blossoms if humans have the skill to
manage technology. Entertainment follows from a shorter work life and longer retirements. Elderly people
are more active than previous generations and demand health care, entertainment, social networks and
communication. They need coaches, presenting huge job openings.

Source: https://scroll.in/article/858766/what-are-the-skills-of-tomorrow-and-how-must-the-worlds-
workforce-adapt

What has the author conveyed regarding the current pattern of Education and Research?
A. Current education and research patterns have no B. It is more inclined towards providing STEM
www.byjusexamprep.com

productivity benefits competences


C. It has somehow lost touch with the need for soft D. It nowhere relates to applying knowledge and
skills finding opportunities offered by technology.
112. Which of the following can be deduced from the above passage?
A. There is a need to identify the evolving job trends B. Acquiring academic knowledge and mapping the
and focus on much demanded cognitive skills set standards should be the ultimate goal.
D. The current education and research patterns
C. Overcoming silo thinking and gaining flexibility at
need to focus more on enhancing soft skills to help
work must be a priority.
tap the growth sectors of the future.
113. As per the above passage, which of the following does not correctly represent the author’s view regarding
education and research?
A. Education and Research must focus on skills that B. It trains people to enable them to perform in future
involve conscious intellectual effort. economic and technological environments
C. One can do away with human improvement which D. STEM is and must be integral part of education
relates to the ability to try new technologies. and research.
114. Which of the following steps must be taken to help education and research become Interdisciplinary and
holistic?
B. Condemning the system that lays much
A. Preparing workers to tackle unknown problems of
importance to cognitive skills and STEM
the future.
competences.
C. Realising the value of social skills and inculcating
D. Enforcing more short-term fiscal planning on
human improvement along with the existing focus on
education programs.
cognitive skills and STEM knowledge.
115. In the above passage, what does the author mean when he says “Much attention is devoted to high-
calibre education and research and for good reason, but demand trends suggest that human factors may
be more essential than normally assumed.”
B. Human factors play a vital role in transforming
A. The government is doing no justice in focusing on
one’s personality and adding stars to one’s
high-calibre education.
capability.
C. High-calibre education and research is not of
D. The demand trends usually remain constant
much importance.
116. Direction: Read the passage given below and answer the questions that follow:
www.byjusexamprep.com

In 1955 Maurice Duverger published The Political Role of Women, the first behavioralist, multinational
comparison of women’s electoral participation ever to use election data and survey data together. His
study analyzed women’s patterns of voting, political candidacy, and political activism in four European
countries during the first half of the twentieth century. Duverger’s research findings were that women voted
somewhat less frequently than men (the difference narrowing the longer women had the vote)

Duverger’s work set an early standard for the sensitive analysis of women’s electoral activities. Moreover,
to Duverger’s credit, he placed his findings in the context of many of the historical processes that had
shaped these activities. However, since these contexts have changed over time, Duverger’s approach has
proved more durable than his actual findings. In addition, Duverger’s discussion of his findings was
hampered by his failure to consider certain specific factors important to women’s electoral participation at
the time he collected his data: the influence of political regimes, the effects of economic factors, and the
ramifications of political and social relations between women and men. Given this failure, Duverger’s study
foreshadowed the enduring limitations of the behavioralist approach to the multinational study of women’s
political participation.

Given below is a possible inference that can be drawn from the facts stated in the passage. You have to
examine the inference in the context of the passage and decide upon its degree of truth or falsity.

“Duverger's study was one of its kind that concluded Women's political participation trends existing in the
mentioned period, stating the ideal behavioralist approach to the multinational study on women’s political
participation, mapping the contexts of changing times.”
A. Definitely true B. Probably true
C. Probably false D. Definitely false
117. Why, according to the passage, the actual findings of Duverger’s study did not conform with the changing
contexts of the then present time?
B. The findings were hindered due to lack of
A. Due to the inaccurate sample data collection that
adequate expertise that affected the extent of
comprised of election and survey data both led to
electoral participation of Women in the period under
the faulty findings of the Duverger’s study.
consideration of the study.
C. The findings were found to be biased as it failed D. The changes were immeasurable, hence were not
to incorporate the unexpected results of the political incorporated
www.byjusexamprep.com

and social relations between women and men.


118. Which of the following statements is NOT TRUE according to the author and as per the information given
in the passage?
B. Duverger's findings were based on the evolving
A. Not all women of Europe exercised their right to
historical processes that shaped electoral activities
vote in the first half of twentieth century.
and thus was not considered much authentic.
C. European women were conservative and there
D. More women exercised their right to vote when
existed complications in political and social relations
they were allowed more time for it.
between men and women.
119. Which of the following could be the primary purpose of writing the passage?
A. To highlight that woman do not cast their vote B. To evaluate a research conducted on the pattern
judiciously. of voting by women
D. To highlight political and social relations between
C. To reconsider the findings of Duverger’s work
women and men
120. What does the author imply when he says “Duverger’s research findings were that women voted
somewhat less frequently than men (the difference narrowing the longer women had the vote)”?
B. Men used to cast their voting rights to the
A. The women did not vote at all
maximum
C. Women possessed the right to vote for a short D. The political views of women were very
span of time conservative
121. Direction: Read the passage given below and answer the questions that follow:

Climate change is of great significance for India, both because of its potentially enormous impacts on the
country and because India can play a decisive part in the global effort to address it. What India can and
should do to address climate change and the support it needs to do so have long been a matter of debate.
Successive Indian governments have taken differing approaches to this in climate negotiations, but one of
the enduring and entirely legitimate planks of our position has been a focus on equity and fairness.

Understanding how India can continue to contribute meaningfully to greenhouse gas (GHG) mitigation,
consistent with equity and fairness, is particularly important given a slew of forthcoming climate summits,
at which we will be expected to deliver an updated pledge. In particular, India will need to decide whether
to join a growing number of countries (over 120 at last count) that have pledged to reach “net zero”
www.byjusexamprep.com

emissions by 2050. The “net zero” idea is inspired by an IPCC report that calls for global net emissions –
GHG emissions minus removal of GHGs through various means, considered at a global aggregate level
— to reach zero by mid-century. This, in turn, builds on a clause in the Paris Climate Agreement, calling
for a balance between sources and sinks of emissions by the second half of the century. It is worth
underscoring that none of this implies that each country has to reach net-zero by 2050. In fact, such an
interpretation flies in the face of equity and fairness.

This emergent trend of net-zero announcements is commendable in that it signals a progressive direction
of travel and has the apparent merit of presenting a simple and singular benchmark for assessing whether
countries are playing their part in addressing the climate challenge. Yet, there are hidden complexities in
this formulation. The use of “net” zero potentially allows countries to keep emitting today while relying on
yet-to-be-developed and costly technologies to absorb emissions tomorrow. Its focus on long-term targets
displaces attention from meaningful short-term actions that are credible and accountable. And, it calls into
question concerns of equity and fairness.

Which of the following cannot be inferred from the passage above?


B. Equity and fairness are two pillars on which the
A. India is yet to commit to the target of achieving
target of achieving net zero emissions is based
net zero emissions by 2050.
upon.
C. Successive Indian Governments have always
D. Climate change is of huge importance of India
focused on achieving meaningful climate change
because India’s stance regarding the same can have
developments while also ensuring equity and
serious global impact.
fairness.
122. Consider the following statements:

I. The emergence of the trend of net zero emissions is a praiseworthy step taken by the nations around
the world and it will certainly bring about positive reforms in the in our battle against climate change.

II. Countries can take advantage of the loopholes present in the complexities surrounding net zero
emissions.

III. The Paris Climate Summit has mandated the countries to achieve net zero emissions by the fall of
2050
www.byjusexamprep.com

As per the above passage, which of these statements is implicit?


A. Only I B. Only II
C. I and II D. II and III
123. Which of the following is not a problem associated with net zero net zero emissions?
A. Countries can use net zero emissions to emit
B. It calls into question the concerns surrounding
greenhouse gases today while relying on upcoming
fairness and equity.
costly technologies to absorb emissions tomorrow.
D. The emerging trend of net zero emissions can
C. The focus on long-term prospects can be
force developing countries into submission regarding
deterrent to the short-term actions and targets that
the urgent need to commit to the cause of climate
could be handy in achieving our long-term goals.
change.
124. As per the passage, which of the following is a major challenge in front of India regarding the greenhouse
gas mitigation?
A. India needs to decide whether to commit to net B. India needs to decide on how to contribute to the
zero emissions by the time next climate summits cause of greenhouse gas mitigation in a telling way
arrive. while also factoring in equity and fairness.
C. India needs to decide which countries to side with D. India needs to decide whether to attend the
during the climate summits. upcoming climate summits or not.
125. Consider the following statement- “It is worth underscoring that none of this implies that each country has
to reach net-zero by 2050. In fact, such an interpretation flies in the face of equity and fairness.”

Which of the following is probably the most appropriate explanation for the above excerpt?
A. Since each and every country needs to reach net-
B. Since each and every country needs to reach net-
zero emissions by 2050, it is expected that countries
zero emissions by 2050, countries have no choice
can ditch the ideas of fairness and equity while focus
but to ditch the ideas of fairness and equity.
on achieving the target.
D. Since each and every country need not reach net-
C. Since each and every country need not reach net-
zero emissions by 2050, it is expected that
zero emissions by 2050, they can be expected to
adherence to equity and fairness can never be taken
adhere to equity and fairness.
care of.
126. Direction: Read the passage given below and answer the questions that follow:
www.byjusexamprep.com

An Environmental Kuznet Curve (EKC) seems to exist between the Covid mortality rate and Sustainable
Development Goal (SDG) scores in India. The United Nations General Assembly had adopted 17 SGDs in
the year 2015 with the mission statement, “A blueprint to achieve a better and more sustainable future for
all people and the world by 2030”. These goals encompass different facets of wellbeing starting from
poverty and hunger to peace, justice and strong institutions. Attainment of these goals is synonymous with
good life that ensures environmental, social and economic sustainability. These goals are a global priority
and developmental policies of the governments are tuned to address the pre-requisites so that the goal
achievements are prioritised and hastened.

The Government of India has fasttracked the SDGs and since 2018, annual SDG scores are being
published for different states and Union Territories (UTs) of the country by Niti Aayog. The commitment of
both central and state governments to SDG is clearly visible as the scores show a rising trend for almost
all units in spite of the Covid pandemic challenges. The overall SDG score for India has gone up from 57
in 2018 to 60 in 2019 and to 66 in 2020, though there is wide discrepancy in scores of the federal units
with Bihar (52) at the bottom and Kerala (74) at the top. The silver lining is that almost all states and UTs
improved their performance in 2020 as compared to 2019 with the exception of Dadra and Nagar Haveli.

A high score is indicative of a higher level of sustainability and an indicator of better management of the
social sector by the administration. A related question is “are high scores better able to control the
mortality rates?” The cumulative Covid mortalities in Indian states and UTs till June 15, 2021 — as
reported by the Covid-19 Data Repository by the Centre for Systems Science and Engineering (CSSE) at
Johns Hopkins University — are compared to their respective SDG scores. As per the JHP data base, the
average cumulative deaths per thousand population varied between 0.08 (Mizoram) to 7.1 (Pondicherry)
in India. To gain a greater understanding, states and UTs are divided into four categories as per their SDG
score and the average death rates are compared. This shows an inverted U relation — as the average
score increases, the death rate increases from low to high and then comes down as shown below (Fig.1).
The death rate is very low (0.2) for the 12 states and UTs with very low SDG score (less than 65), peaked
at 0.95 for ones with scores between 65 to 70, then started decreasing to 0.85 for those with scores
between 70 to 74 and then to 0.5 for scores above 74. Though there is discrepancy across states and
UTs, the average picture for groups confirms the ability of high-scorer states/UTs to have been better able
to control the pandemic deaths. This relation becomes all the more pronounced when the 95 per cent
confidence interval of predicted mortality is plotted against the difference in scores of the states between
2018 and 2020.
www.byjusexamprep.com

States attaining a high score in 2020 compared to their achievement in 2018, depict much lower predicted
deaths and this relation, though has an inverted u-shape like the previous picture, is sharper and goes
down monotonically for score differences more than 3.5. Thus, investing in sustainable development goals
seems to be helping the states to better manage the pandemic, which is an indicator of sustainability.

According to the author, which of the following is a plausible explanation for low COVID mortality rates in
some states?
A. Achieving sustainable development goals have
B. Achieving sustainable development goals has
allowed some states to take control of the pandemic
nothing to do with better pandemic management.
much effectively.
C. Achieving sustainable development goals has D. Achieving sustainable development goals has
shifted our focus from the worsening pandemic allowed the states to leverage their low mortality rate
situation. as a bargaining chip in opening up the economy.
127. According to the author, what is the correlation between covid mortality rates and sustainable
development goals score?
A. There is a huge discrepancy across various
states and union territories as far as the correlation B. As soon as the average SDG score increases, the
between covid mortality rates and sustainable death rate falls.
development goals (SDG) is concerned.
D. While there is discrepancy across various states
C. The COVID-19 mortality rate is directly
and union territories, it can be validated that a low
proportional to the average SDG score.
mortality rate can be attributed to a good SDG score.
128. Which of the following statements can be inferred from the passage?
A. SDG ensures having a good life, free from B. The Government of India has laid strong
economic and social hardships. emphasis on the completion of SDG.
C. Various global governing bodies have pressured D. The SDG have influenced policymaking in various
national governments to make the SDG a priority. developing nations around the world.
129. Which of the following statements, if true, strengthens the author’s argument that achieving SDG goes a
long way in ensuring low mortality rates in states battling with the pandemic?
A. Achieving SDG for this year will take a lot of the B. SDG is not the only parameter that can or should
states such as Bihar, Uttar Pradesh, etc. who are be used to ensure low mortality rates in states
already grappling with the pandemic. suffering the most from the pandemic.
www.byjusexamprep.com

C. A recent study has revealed that those states D. Achieving SDG is not as easy as it sounds;
involved the most in achieving the SDG have however, rewards are worth the effort.
achieved exceptionally low levels of mortality rates.
130. According to the passage, which of the following statements is incorrect?
A. The SDGs encapsulate various facets of life that
B. Achieving the SDGs must be prioritized at all
are required to live a sustainable life in the present
costs by the nations.
as well as in the future.
D. SDGs are a great measure to gauge a nation’s
C. The SDGs are a nation’s progress towards
progress in terms of COVID mortality and
achieving self-sustained economy and ecology.
vaccination rates.
131. Direction: Read the passage given below and answer the questions than follow:

On July 20, GoI made two eye-catching observations in Parliament about the second Covid wave. It said
states had reported that there were no deaths due to medical oxygen shortage, or due to lack of testing
and treatment. This is plainly inconsistent with collective experience. April 2021 to June was the worst
phase in the Covid trajectory, and large parts of the country were locked down as healthcare infrastructure
was overwhelmed. GoI’s data showed that 2.35 lakh deaths, or 56% of all Covid fatalities till date, took
place in those three months. It beggars belief that not one death was due to the oxygen supply crisis. The
problem is states and hospitals are probably using record-keeping protocols to avoid grim facts.

A larger, related question is whether India is undercounting Covid fatalities. There’s been plenty of
research on this. The most commonly used source now is the Civil Registration System (CRS), a record of
deaths maintained by states. While this system does not offer data on just Covid deaths, it provides a
sense of the “excess mortality” in 2020-21 that can be assumed to have been influenced largely by the
pandemic. An alternative indicator is the GoI’s Sample Registration System (SRS), a demographic survey,
which is available till only 2019

Arvind Subramanian, a former GoI chief economic advisor, and his associates, recently estimated that,
under some assumptions, using CRS shows that excess mortality between April 2020 and June 2021 was
3.4 million. There’s an argument against using CRS, which is that it’s not as accurate as GoI’s SRS. That’s
partly true. Data of 2019 shows that in the southern states there’s no mismatch between CRS and SRS.
But many other states’ CRS underestimate deaths.
www.byjusexamprep.com

Deaths are often used as a proxy for governance in political rhetoric. Politics over deaths provides
perverse incentives to undercount and underplay the severity, helped by the fact that Covid death
registration guidelines are too tight. That’s deeply unfair to families of victims of the second wave. They
deserve an honest answer. The way out is for GoI to do a proper SRS, a survey that’s been in place for 50
years. GoI’s data shows that so far there are 4.18 lakh Covid fatalities, which is 1.34% of people who
tested positive. But if sero surveys indicate that the number of people infected is far larger than test data,
what’s the real scale of fatalities? The answer can only come through a nationwide demographic survey.

Which of the following, if true, strengthens the idea that there is a pressing need for a nationwide
demographic survey to gauge the accurate number of deaths occurring due to the COVID-19 pandemic?
A. Various independent and unbiased institutions
B. The Government of India has tried hard to gauge
have pointed out that undercounting of deaths in
the exact number of the fatalities associated with the
India occurring due to public health disasters have
pandemic and so far, they have succeeded partially.
been a serious problem since ages.
C. Various petitioners have pointed out that we do D. The government of India is seriously considering
not need a nationwide demographic survey as long the requirements for the nationwide demographic
as the undercounting does not occur. survey.
132. Which of the following, if true, weakens the idea that there is a pressing need for a nationwide
demographic survey to gauge the accurate number of deaths occurring due to the COVID-19 pandemic?
A. A nationwide demographic survey is only going to B. A nationwide demographic survey will surely help
increase the government’s expenditure, which can us in getting a clearer picture regarding the number
be put to better use in times of crisis like this. of deaths occurring due to the pandemic.
C. Recently, a similar nationwide survey was D. The government does not look interested in
successfully carried out by the U.S. carrying out such surveys.
133. “Deaths are often used as a proxy for governance in political rhetoric. Politics over deaths provides
perverse incentives to undercount and underplay the severity, helped by the fact that Covid death
registration guidelines are too tight.”

What is the author trying to convey through this statement?


B. It is not like the government is running away from
A. The governments have always tried to shy away
taking all the responsibility for the deaths caused by
from all the negativity surrounding the deaths of
the pandemic. It’s just that this matter should not be
people from public health crisis.
politicized.
www.byjusexamprep.com

C. Deaths have always been considered a substitute D. The government has no hidden motive to
for politics, marked by the morally wrong incentives undercount the deaths caused by the pandemic.
to undercut the casualties caused by the pandemic
and helped by the strictness of COVID-19 death
registrations.
134. According to the given passage, which of the following statements is not true?
A. The Government of India is deliberately trying to B. Both Civil Registration System (CRS) and Sample
undercount the number of casualties associated with Registration System (SRS) can be trusted to some
the COVID-19 pandemic. extent.
D. A nationwide demographic survey is needed to
C. Most of the people died during the deadly second
weed out the problem of misreporting of cases and
wave of the pandemic probably went unrecorded.
gauge the actual scale of the fatalities.
135. Which of the following statements is true with respect to the passage?
A. The CRS is just as reliable as SRS. B. The CRS is much more reliable than the SRS.
D. The CRS and the SRS are reliable in some
C. The CRS is much less reliable than the SRS.
instances and not reliable in the others.
136. Direction: Read the data carefully and answer the questions.

A survey is conducted on 1600 civilians of Delhi to their suggestions on the topic ‘ How COVID 19 Can Be
Stopped’. One-fourth of the total persons gave the only suggestions of social distancing. 10% of the total
persons gave the only suggestion of wearing a mask. 12% of the total persons gave the only suggestion
of using Sanitizer. 8% of the total persons gave the only suggestion of ‘Washing hands with antibiotic
soap’. 20% of the total persons gave the only suggestions of vaccines. 8% of the total persons gave
suggestions of social distancing and wearing a mask only. 6% of the total persons gave the only
suggestion of social distancing, wearing a mask and vaccine also. 5% of the total persons gave the
suggestion of wearing a mask, washing hands with antibiotic soaps and vaccine only and 6% of the total
persons gave all the five suggestions.

Total people out of the surveyed people who gave suggestion of using Sanitizer only, washing hands with
anti – biotic soaps only and vaccine only together are approximately what percent more or less than total
no. of surveyed people of the town who gave suggestion of only social distancing and only wearing a
mask together?
A. 10% B. 14 %
www.byjusexamprep.com

C. 12% D. 18%
E. 16%
137. Find total no. of people out of surveyed people who gave at most two suggestions?
A. 1164 B. 1162
C. 1184 D. 1328
E. 1188
138. Find total no. of people out of surveyed people who gave at least two suggestions?
A. 1200 B. 1400
C. 800 D. 400
E. 2000
139. Find ratio between total no. of people out of surveyed people of town who gave suggestion of wearing a
mask, washing hands with soap and vaccine only to total people of town who gave suggestion of all the
five ideas?
A. 6 : 5 B. 5 : 4
C. 5 : 7 D. 5 : 9
E. 5 : 6
140. Total no. of persons out of surveyed persons who gave suggestion of vaccine only is what percent more
than the no. of persons out of surveyed persons who gave suggestion of all social distancing, wearing a
mask & vaccine only?
A. 120% B. 140%
C. 160% D. 100%
E. None of these
141. Direction: Study the following information carefully and answer the questions given below.

Anup has started a business of electronics items in which he purchased mouse, printer, scanner, laptop,
CPU, monitor, playstation, and Ipad. The cost of a laptop is 5 times the cost of a scanner. The cost of a
scanner is 3/5 of the Ipad. The cost of an Ipad is 3 times the cost of a printer. The cost of a CPU is 2000
more than the cost of an Ipad. The price of a printer is 5 times the price of a mouse. The cost of a monitor
is 30% more than Ipad. The cost of a Playstation is half of the cost of an Ipad and CPU together and the
cost of a mouse is 1000.
www.byjusexamprep.com

What will be the total profit of Anup if he sell one printer at 10% profit and one Scanner at 15% profit
together.
A. 1550 B. 1850
C. 1900 D. 2100
E. None of these
142. Find the total cost of 2 playstations, 1 laptop and 3 scanner together.
A. 104000 B. 114000
C. 124000 D. 100000
E. None of these
143. Find the ratio of cost of a scanner to cost of a monitor.
A. 1 : 2 B. 2 : 1
C. 2 : 3 D. 3 : 2
E. None of these
144. If Anup got 10% discount on cost while purchasing laptop and he marked 20% above the cost price and
sold the laptop in 5% additional discount then find the total profit earned by Anup.
A. 16% B. 14%
C. 18% D. 12%
E. None of these
145. Price of playstation is how much percentage more than the price of scanner ?
A. 77.77% B. 55.66%
C. 66.66% D. 43.75%
E. None of these
146. Direction: Study the given information carefully and answer the following questions.

There are four e-commerce companies namely Amazon, Flipkart, Myntra & Ajio which sell both Electronic
& Non-electronic items. Total Non-electronic & Electronic sold by Flipkart is 25% more than total Non-
electronic & Electronic sold by Myntra. An average number of total Non-electronic & Electronic sold by
Amazon & Myntra is 3800. Total Non-electronic & Electronic sold by Amazon is 20% more than total Non-
electronic & Electronic sold by Ajio. Percentage of Non-electronic sold out of total Non-electronic &
Electronic sold by each of Amazon, Flipkart, Myntra & Ajio is 30%, 16%, 20% & 40% respectively. Non-
electronic sold by Myntra are 280 less than the Non-electronic sold by Amazon.
www.byjusexamprep.com

Non-electronic items sold by Flipkart & Ajio together is what percent of total non-electronic & electronic
items sold by Myntra?
A. 50% B. 40%
C. 25% D. 20%
E. 60%
147. Find average number of electronic items sold by all four companies.
A. 2450 B. 2930
C. 3220 D. 3020
E. 4220
148. Non-electronic sold by Amazon & Ajio together are what percent more or less than electronic sold by Ajio?
A. 42.25% more B. 32.75% less
C. 21.33% more D. 26.67% more
E. 38.33% less
149. What is the ratio of total Non-electronic & Electronic sold by Amazon & Ajio together to total Non-
electronic & Electronic sold by Flipkart & Myntra together?
A. 13 : 17 B. 13 : 15
C. 11 : 13 D. 11 : 17
E. 11 : 15
150. Find difference between average of total Non-electronic & Electronic sold by Flipkart, Myntra & Ajio and
total Non-electronic sold by all four companies?
A. 120 B. 150
C. 140 D. 210
E. 230
www.byjusexamprep.com

Solutions

1. B
Sol. The correct answer is option B.
Refer to the eighth and ninth paragraphs of the passage: “One explanation for their survival may be that
they were placed within an underground room, Goddio said, noting a possible funerary connotation.

It is within an area where Goddio and his team of archaeologists have discovered a sizeable tumulus (a
mound raised over graves) – about 60 metres long by 8 metres wide – and sumptuous Greek funerary
offerings.”
From these lines, it can be inferred that the fruit basket can be used to shed light on the funerary practices
of the Greek civilization, rather than the Roman one, which finds no mention in the passage. Thus, option
B is correct and option D is wrong.
Colonialism has not been mentioned in the passage, thus making option A irrelevant.
Refer to the following quote from the fifth paragraph of the passage: “Colossal statues were among
treasures from an opulent civilisation frozen in time.” The use of the word ‘opulence’, meaning great
wealth or luxuriousness, negates option C, since the line makes it clear that Thonis-Heracleion was a
prosperous city.
2. C
Sol. The correct answer is option C.
Refer to the second and fourth paragraphs of the passage: “…the city disappeared beneath the waves
in the second century BC, then sunk further in the eight century AD, following cataclysmic natural
disasters, including an earthquake and tidal waves. But the vast site in Aboukir Bay near Alexandria was
forgotten until its re-discovery by the French marine archaeologist Franck Goddio two decades ago,
in one of the greatest archaeological finds of recent times.” This makes option C the correct answer.
Options A and D have not been mentioned in the passage and are thus irrelevant to it.
As for option B, despite Alexander setting up the city of Alexandria, which replaced Thonis-Heracleion as
the largest port city of Egypt, he did not destroy the city. It simply fell victim to natural calamities.
3. A
Sol. The correct answer is option A.
In this passage, the author is trying to inform the readers of the latest findings at the excavated site, as
can be deciphered from the lines, “Wicker baskets filled with fruit that have survived from the 4th century
www.byjusexamprep.com

BC and hundreds of ancient ceramic artefacts and bronze treasures have been discovered in the
submerged ruins of the near-legendary city of Thonis-Heracleion off the coast of Egypt.” Thus, option A is
the correct answer.
The passage does not mention anything about the site being a sinkhole (a depression or hole in the
ground caused by some form of collapse of the surface layer). Simply that the city had remained
submerged until its latest discovery. Thus, option B is incorrect.
Option C is incorrect because the passage clearly states that “the city disappeared beneath the waves
in the second century BC, then sunk further in the eight century AD, following cataclysmic natural
disasters, including an earthquake and tidal waves.”
Option D is incorrect since the passage is objective in its reporting of the latest findings rather than critical
of the excavators.
4. D
Sol. The correct answer is option D.
Refer to the following quote from the third paragraph of the passage: “…was for centuries Egypt’s largest
port on the Mediterranean…”
Refer also to the following quote from the last paragraph of the passage: “…controlled the entrance to
Egypt at the mouth of the Canopic branch of the Nile…”
This shows that the city was strategically important to ancient Egypt, making option D the correct answer.
Option A is wrong since the city is a part of Egypt, and is thus, not a part of Europe.
Options B and C are not mentioned in the passage and are thus irrelevant to it.
5. D
Sol. The correct answer is option D.
Sumptuous: impressive in a way that seems expensive.
Cheap: costing little money or less than is usual or expected.
Opulent: expensive and luxurious.
Lavish: large in quantity and expensive or impressive.
Luxurious: very comfortable and expensive.
6. A
Sol. The correct answer is option A.
Refer to the fifth and sixth paragraph of the passage: “The unprecedented heat events…had disastrous
effects on people, plants, and animals.
As the environment continues to warm due to human-caused global heating,….the compounding
catastrophes that have plagued the west this summer will persist into the future, continuing to wreak
www.byjusexamprep.com

havoc on ecosystems, infrastructure, and agriculture.”


Thus, the passage clearly tries to pinpoint the seriousness of the impact continued global heating will have
on the environment and its components: plants, animals, and people.
Option B stands in direct contrast to the points mentioned in the passage.
Options C and D have not been outlined in the passage, making them irrelevant to it.
7. C
Sol. The correct answer is option C.
The tone of the passage clearly implies the author’s clear disdain for the environmental costs of global
warming. As such, the author will not likely be using the premise of increased fishing opportunities as an
argument against global warming. The passage expresses concern for the dead sea creatures rather
than see an economic opportunity in the event. Thus, option C is the correct answer.
The points made in options A, B, and D all relate to the disastrous effect of global warming on
ecosystems, agriculture, and infrastructure, and will thus be used by the author to strengthen the case
against the impact of global warming, rather than weaken it.
8. A
Sol. Option A is the correct answer.
The passage uses the example of the death of the sea creatures due to the heatwaves to outline the
adverse impact of global warming on human life and property. This can also be seen in the following quote
from the second last paragraph of the passage: “…continuing to wreak havoc on ecosystems,
infrastructure, and agriculture.” Thus, option A is the correct answer.
Option B is automatically supported by the passage.
Option C is supported by the following quote from the fourth paragraph of the passage: “Scientists expect
the impact will have a trickle-down effect on the ecosystem and the other animals that rely on those
that died for food and habitat.”
Option D is supported by the following quote from the sixth paragraph of the passage: “As the
environment continues to warm due to human-caused global heating…”
9. A
Sol. The correct answer is option A.

The term ‘trickle-down effect’ is used to refer to a situation in which something that starts in the high parts
of a system spreads to the whole of the system. In this situation, the heatwaves have killed billions of sea
creatures, because of which creatures depending on the dead animals for food and habitat will also suffer,
causing a trickle-down and spread of the disaster throughout the ecosystem. Since the destruction of the
www.byjusexamprep.com

larger ecosystem occurs as a consequence of the death of the sea creatures, option A will be the best
answer.
10. B
Sol. The correct answer is option B.
When one is ‘exposed to the elements’, one is likely to be influenced or harmed by a particular thing. In
the passage, the phrase is used to imply that the sea creatures were vulnerable to and at risk of being
harmed by the changes in the atmospheric conditions. The phrase closest to this meaning is ‘in harm’s
way’, making option B the correct answer.
11. B
Sol. The correct answer is option B.

Refer to the following quote from the third paragraph of the passage: “A study on "The Impact of COVID
19 and Industry 4.0 on Future of Work for Women, An Insight from Formal Sector in India" underscores
that alternative work arrangements in the gig economy have the potential to absorb more women and
increase their participation in the workforce.”
This statement, makes option B the correct answer while negating options A, C, and D.
12. A
Sol. The correct answer is option A.
Refer to the following quote from the third paragraph of the passage: “…However, concerted efforts are
needed to understand how new technologies are impacting specific industries and to address
challenges facing women in entering or remaining in the workplace...”
Refer to the following quotes from the last paragraph of the passage: “At the same time, women's
employment in the finance and accounting divisions may moderately change due to the adoption of
new technology.”
“…reskilling will be crucial to absorb more women in the world of work in India…”
It can be clearly inferred from these three quotes that new technology may pose challenges to women
being absorbed into the workforce, and their impact should be well assessed and women reskilled before
they can start actively participating in the workforce.
These quotes also negate option B as a possible inference.
Option C is incorrect since ‘old technologies’ have not been mentioned in the passage, which continues
reflecting on how new technologies are impacting industries.
Option D cannot be inferred since although India has been mentioned in the passage with reference to gig
economy, nowhere does the passage state developing economies in general, or that unskilled labour force
www.byjusexamprep.com

might become a factor for deterrence. What the passage does try to do is strike a note of optimism by
stressing on reskilling rather than rejecting.
13. C
Sol. The correct answer is option C.
Refer to the following quote from the second paragraph of the passage: “Gig economy is a labour market
characterised by the prevalence of short-term contracts or freelance work as opposed to permanent jobs.”
14. D
Sol. The correct answer is option D.
The author of the passage does not let personal opinion influence the passage. The passage simply
states the observations made by a study, and how those observations may impact real scenarios. Thus,
the tone of the passage is objective, meaning (of a person or their judgement) not influenced by personal
feelings or opinions in considering and representing facts.
Cynical: believing that people are motivated purely by self-interest; distrustful of human sincerity or
integrity.
Critical: expressing adverse or disapproving comments or judgements.
Scornful: feeling or expressing contempt or derision.
15. B
Sol. The correct answer is option B.
Since a ‘gig economy’ is based on contractual and freelance work as opposed to permanent jobs, it
automatically brings in the question of job security, implying that the employees do not have an assurance
that they will have their job in the foreseeable future irrespective of outside factors.
16. A
Sol. The meaning of the following words is:

Expository: intended to explain or describe something

Controversial: giving rise or likely to give rise to controversy or public disagreement

Abstract: existing in thought or as an idea but not having a physical or concrete existence

Narrative: a spoken or written account of connected events; a story


www.byjusexamprep.com

The passage describes protein and how new AI tool Alphafold is crucial in studying protein. Thus, option A
is the most apt answer.
17. C
Sol. The passage states, “the software looks likely to be massively useful, helping researchers spot
possibilities and dead ends more quickly and letting them take on projects they would otherwise have
steered clear of”, especially in the study of proteins. Thus, option D is the most appropriate choice.
18. C
Sol. The passage states that “as proteins that flex like springs, spin like wheels, extend like pistons and crank
like ratchets, that turn sunlight into chemical energy, that build, demolish and recycle all the components of
life and do more besides”. This makes option A, B and D false. Only option C is correct.
19. C
Sol. The passage states “To understand the structure of a protein in detail, scientists have had to make
comparatively large amounts of it, coax those molecules into forming a crystal (or, more recently, flash‐
freeze them) and bombard the sample with xrays (or, if it is a frozen one, electrons). These procedures
take time, money and effort. Only a tiny fraction of the proteins whose sequences are known have been
studied this way.”

This makes option C the correct answer.


20. B
Sol. The correct answer is option B.

Both similes and metaphors are used to make comparisons, the difference between similes and
metaphors comes down to a word. Similes use the words like or as to compare things—“Life is like a box
of chocolates.” In contrast, metaphors directly state a comparison—“Love is a battlefield.”

Out here, a direct comparison is made between protein structure and space. Thus, option B is correct.
21. A
Sol. Throughout the passage, the author shows how we are optimizing our life for things which is ridiculous.
Our goal is optimizing our life around it. For us, scrolling through social media is more important that
having a normal meal, and we optimize our life around it. Thus, option A is the correct choice.
22. B
Sol. Instead of enjoying a meal, the author observes that people who go to Sweetgreen eat because they need
to eat to survive.
www.byjusexamprep.com

The passage states that one would not enjoy a meal so that one could go on working.

Thus, option B is the correct choice.


23. B
Sol. As per the passage progress is, “what you do when you’ve gotten ahead a little bit, when you want to get
ahead some more”. This can be described as hamster-wheel situations which means “any situation that
seems to be endlessly without goal or achievement as there is no end”. Thus, option B is the correct
choice.
24. C
Sol. The passage states that “the chopped salad is engineered to “free one’s hand and eyes from the task of
consuming nutrients, so that precious attention can be directed toward a small screen, where it is more
urgently needed, so it can consume data: work email or Amazon’s nearly infinite catalogue or Facebook’s
actually infinite News Feed. On today’s terms, what Buchanan is describing is the good life”. Thus, option
C is the correct choice.
25. C
Sol. The given statement tells us to get rid of vulnerability instead of facing it head-on and getting over it. So, in
order to find the reversal of this statement, we need to pick an answer option that urges us to be strong in
the face of adversity and not to give in to the things which you do not believe in or which does not make
any sense to you. The above idea is best captured by option C and hence, it is the right answer.
26. B
Sol. Optimistic: Hopeful and confident about the future

Anxious: feeling or showing worry

Compassionate: feeling or showing sympathy and concern for others

Critical: expressing adverse or disapproving comments or judgements

The author is worried about the future of the climate as increase of red algae on mountain tops signifies
climate change. Thus, option B is the correct choice.
27. C
Sol. Option A is not mentioned in the passage. Thus, cannot be taken as answer.

Option B is false in nature because “red snow algae have been known for a long time”.
www.byjusexamprep.com

Option D is false as snow algae is not changing due to climate change. It’s increase is a result of climate
change, not the cause of it.

The passage states “However, when the snow algae grow prolifically and are exposed to strong solar
radiation, they produce red-coloured pigment molecules known as carotenoids, which act as a sunshield
to protect their chlorophyll.” Thus, option C is the correct choice.
28. D
Sol. The meaning of the following words are:

Garish: obtrusively bright and showy; lurid

Drab: Lacking brightness or interest; drearily dul

Calamitous: involving calamity; catastrophic or disastrous

Heinous: (of a person or wrongful act, especially a crime) utterly odious or wicked

Gaudy: extravagantly bright or showy

Thus, option D is the correct answer.


29. B
Sol. The phrase ‘status quo’ means: the existing state of affairs.

Thus, option B is the correct answer.


30. D
Sol. The passage states that “. By trapping more of the Sun's heat, the snow melts even faster, allowing the
algae to proliferate further. "There is a runaway effect in which the algae melt their preferred habitat," says
Benning. "It’s as if they are destroying their own house." Thus, option D is the correct answer.
31. B
Sol. The Ramgarh Vishdhari wildlife sanctuary has recently received a nod from the National Tiger
Conservation Authority’s (NTCA) technical committee to become the 4th Tiger reserve of Rajasthan.
The remaining three Tiger reserves of Rajasthan are: (i) Mukundra Hills Tiger Reserve (MHTR) in Kota, (ii)
Sariska Tiger Reserve (STR) in Alwar and (iii) Ranthambore Tiger Reserve (RTR) in Sawai Madhopur.
Rajasthan is abundant in wildlife and it's conservation with the help of various Wildlife sanctuaries and
Tiger Reserves.
www.byjusexamprep.com

32. C
Sol. Every year, 29th of July is celebrated as the International Tiger Day or World Tiger Day to raise
awareness about the conservation of tigers and the dangers posed to them. The theme for International
Tiger Day 2021 is 'Their survival is in our hands'. The theme basically highlights the important role in
protecting the natural habitat of tigers and maintaining a balanced ecosystem across the globe.
33. B
Sol. During the St. Petersburg declaration of 2010, held in Russia, it was decided to celebrate 29th July as
Global Tiger Day across the world, which has since been celebrated to spread and generate awareness
on tiger conservation. India has already achieved the target of doubling the Tigers population in the
country by 2022, set by the St. Petersburg declaration. India currently has 2,967 tigers which was nearly
1,400 in 2006.
34. D
Sol. National Chambal Gharial Wildlife Sanctuary is located on the tri-junction of Rajasthan, Madhya
Pradesh and Uttar Pradesh. It is a 5,400 km² (in area) tri-state (Rajasthan+Madhya Pradesh+Uttar
Pradesh) protected area in northern India for the protection of the Critically Endangered gharial, the red-
crowned roof turtle and the Endangered Ganges river dolphin.
35. C
Sol. The National Tiger Conservation Authority (NTCA) is a statutory body which works under the jurisdiction of
the Ministry of Environment, Forests and Climate Change (MoEF & CC). It was established in 2005
following the recommendations of the Tiger Task Force. It was constituted under enabling provisions
of the Wildlife (Protection) Act, 1972, as amended in 2006, for strengthening tiger conservation, as per
powers and functions assigned to it.
36. A
Sol. The 'Breathe Applied Sciences', a Bengaluru-based start-up, has recently received the National Award
for Recycling Carbon Technology 2021 by the Department of Science and Technology, Government of
India. It has been incubated at the Jawaharlal Nehru Centre for Advanced Scientific Research (JNCASR),
Bengaluru which has recently developed efficient catalysts and methodologies for the conversion of
carbon dioxide (CO2) to methanol and other chemicals.

37. C
Sol. Every year, 11th May is being celebrated as National Technology Day. It is a day to commemorate the
scientific and technological achievements of Indian scientists, engineers and eminent technocrats across
www.byjusexamprep.com

the nation. The day was named by the former Prime Minister Atal Bihari Vajpayee. The greatest
significance of this day is that India successfully tested nuclear bombs in Pokhran on May 11, 1998.
Every year, National Technology Day has a certain motto, objectives and theme. The theme for National
Technology Day 2021 is 'Science and Technology for a Sustainable Future'.
38. A
Sol. In the field of Carbon Technology and it's processing, the term CCUS stands for Carbon Capture,
Utilization and Sequestration. It is basically a process that captures carbon dioxide emissions from
sources like coal-fired power plants and either reuses or stores it so it will not enter the atmosphere.
39. C
Sol. The International Conference on Nanoscience and Nanotechnology (ICONN) is a biennial program which
is being hosted under the aegis of Nano Mission, Department of Science and Technology (DST),
Government of India. The International Conference on Nanoscience and Nanotechnology (ICONN 2021)
was the sixth edition of this program which was virtually concluded at SRM Institute of Science and
Technology (SRMIST), Kattankulathur, Tamil Nadu.
40. A
Sol. The Nano Mission (Nano Science and Technology Mission – NSTM) was launched by the
Government of India in 2007 under the Department of Science and Technology, Ministry of Science and
Technology, Government of India. Hence, only option A is incorrect regarding the Nano Mission or
Nanotechnology. A total fund of Rs. 1000 crores has been allocated to this mission by the Ministry of
Science and Technology.
41. A
Sol. Recently, the Asian Development Bank (ADB) and the government of India has signed a $484 million
loan for the Tamil Nadu Industrial Connectivity Project. The loan is to improve transport connectivity and
facilitate industrial development in the Chennai - Kanyakumari Industrial Corridor (CKIC) in the state of
Tamil Nadu.

42. C
Sol. Four states; West Bengal, Odisha, Andhra Pradesh and Tamil Nadu are covered under the East Coast
Economic Corridor (ECEC) project. Visakhapatnam to Chennai segment of this Corridor has been taken
as phase-1. Vizag-Chennai Industrial Corridor (VCIC) is the first coastal economic corridor in the
country. It covers more than 800 km of Andhra Pradesh’s coastline and is aligned with the Golden
Quadrilateral. It also plays a critical role in the “Act East Policy” of India.
www.byjusexamprep.com

43. D
Sol. The Government of India has recently approved the development of the five industrial corridor projects
which will be implemented through National Industrial Corridor Development and Implementation
Trust (NICDIT). These five industrial corridors are: East Coast Economic Corridor (ECEC), Amritsar-
Kolkata Industrial Corridor (AKIC), Bengaluru-Mumbai Economic Corridor (BMEC), Chennai-Bengaluru
Industrial Corridor (CBIC) and Delhi-Mumbai Industrial Corridor (DMIC).
44. B
Sol. India has recently signed an agreement extending a USD 100 million Line of Credit (LOC) to Sri Lanka for
projects in the Solar Energy Sector. This bilateral agreement was signed between the Government of
Sri Lanka and the Export-Import (EXIM) Bank of India. EXIM Bank is a specialized financial institution,
wholly owned by the Government of India.
45. B
Sol. The Asian Development Bank (ADB) is a regional development bank established on 19th December 1966
which is headquartered in Manila, Philippines. It has currently 68 nations as it's member in which 49
are Asian nation's. Hence, statement II is incorrect regarding ADB. Whereas Japan and the United
States of America (USA) are the largest shareholders (each with 15.6% of total shares) in ADB.
46. B
Sol. Recently, the maiden Indian Navy - European Union Naval Force (IN-EUNAVFOR) Exercise was
successfully conducted in Gulf Of Aden. Along with Indian Navy, the naval forces of Italy, Spain and
France also participated in the exercise. The main aim of the exercise is to enhance and strengthen their
war-fighting skills and their ability as an integrated force to promote peace, security and stability in the
maritime domain.

47. C
Sol. For the first time in the year 2018, the Indian Navy inaugurated the Information Fusion Centre (IFC) for the
Indian Ocean Region (IOR) at Gurugram, Haryana which is also the Indian Navy’s Information
Management and Analysis Centre (IMAC). The IFC-IOR is established with the vision of
strengthening maritime security in the region, by building a common coherent maritime situation
picture and acting as a maritime information hub for the region.
48. A
Sol. The Indian Navy and European Union Naval Force are a part of annual SHADE (Shared Awareness and
Deconfliction) meetings held annually at Bahrain. SHADE or Shared Awareness and Deconfliction is
www.byjusexamprep.com

an international operational counter piracy platform, convened in Bahrain which aims to encourage de-
conflict operations amongst counter-piracy actors in the Gulf of Aden, the Gulf of Oman and the
Western Indian Ocean.
49. B
Sol. Approximately, 11% of the world's total seaborne petroleum passes through the Gulf of Aden on its
way to the Suez Canal or to regional refineries. The Gulf of Aden is a very prominent waterway for
shipping, especially for Persian Gulf oil, making it an integral waterway in the world economy.
50. D
Sol. It's a very well-known fact that the Indian Navy and European Union Naval Force are regularly involved in
naval assistance and different deployments under the charter of World Food Programme (UN WFP). The
United Nations World Food Programme (UN WFP) works closely with the prominent Rome-based UN
agencies like the Food and Agriculture Organization (FAO) & International Fund for Agricultural
Development (IFAD). All of the above mentioned options are correct regarding the United Nations World
Food Programme (UN WFP).
51. B
Sol. Every year, 11th July is observed as the World Population Day. It is the day recommended by the
United Nations Development Programme (UNDP) in 1989 to focus attention on the urgency and
importance of population issues. The UNDP was inspired by the awareness and public interest that was
created by "Five Billion Day" on 11th July 1987 when the world's population reached 5 billion.

52. D
Sol. World Population Day is observed every year with a common objective to control the overpopulation and
difficulties created by it by raising awareness among people across the globe. Every year, this important
day is observed with a different motto or theme or goal to achieve in coming years. Theme for World
Population Day 2021 is "Rights and Choices are the Answer: Whether baby boom or bust, the
solution to shifting fertility rates lies in prioritising all people’s reproductive health and rights."
53. B
Sol. The United Nations Population Fund (UNPF) was initially established as a trust fund in 1967 and
began operations in 1969. It is a subsidiary organ of the United Nations General Assembly and works as a
sexual and reproductive health agency. The UN Economic and Social Council (ECOSOC) establishes its
mandate.
54. C
www.byjusexamprep.com

Sol. The United Nations Population Fund’s (UNFPA) flagship report, called State of World Population Report
2021, was recently released and titled as 'My Body is My Own'. This is the first time a United Nations
report has focused on bodily autonomy, defined as the power and agency to make choices about your
body without fear of violence or having someone else decide for you.
55. D
Sol. The Uttar Pradesh government has recently released 'Uttar Pradesh Population Policy 2021-2030' to
reduce infant and maternal deaths in a time-bound manner. This policy primarily aims to decrease the
total fertility rate from 2.7 to 2.1 by 2026 and 1.7 by 2030; to increase modern contraceptive
prevalence rate from 31.7% to 45% by 2026 and 52% by 2030 & to decrease maternal mortality rate
from 197 to 150 by 2026 and 98 by 2030. Therefore, all the above mentioned options are correct
regarding the 'Uttar Pradesh Population Policy 2021-2030'.
56. A
Sol. The Israeli firm 'NSO Group' was constituted in 2010 which later on developed the famous Pegasus
spyware. Some reports have claimed that Pegasus spyware has secretly been used by the Government of
India on some famous Public figures and agencies. Pegasus is basically a type of malicious software or
malware classified as a spyware.

57. B
Sol. The 12th and latest edition of 'India Security Summit' with the theme "Towards New National Cyber
Security Strategy" was held in New Delhi on 28th August, 2019. Some major and critical issues such
as protection of critical national infrastructure, emerging cyber threats: incidents, challenges and
responses were discussed during the conference. Challenges & solutions of cybersecurity faced by
different agencies and prominent institutions were also discussed during the conference.
58. B
Sol. A 1984-batch Indian Administrative Service officer of Andhra Pradesh cadre, Ajay Prakash Sawhney, is
currently serving as the Secretary of the Ministry of Electronics and Information Technology (MeitY),
Government of India. The Ministry of Electronics and Information Technology (MeitY) was separated
from the Ministry of Communications and Information Technology on 19 July 2016 as a standalone
ministerial agency responsible for IT policy, strategy and development of the electronics industry.
59. B
Sol. The Budapest Convention on Cybercrime came into force on 1st July 2004. It is an international treaty
that seeks to address Internet and computer crime (cybercrime) by harmonizing national laws, improving
www.byjusexamprep.com

investigative techniques, and increasing cooperation among nations.


60. A
Sol. The International Telecommunication Union (ITU) is a specialized agency of the United Nations for
information and communication technologies. It was founded in 1865 to facilitate international connectivity
in communications networks. It is headquartered in Geneva, Switzerland.
61. C
Sol. The International Food Policy Research Institute (IFPRI) has recently released the 'Global Food
Policy Report 2021' which explicitly published the impacts of rising poverty and reduced livelihoods are
reflected clearly in rising levels of food insecurity and decreasing diet quality. The 'State of Food Security
Nutrition in the World 2021 (SOFI)' report was recently published which also studied the impact of COVID-
19 Pandemic on Food, income and malnutrition.

62. D
Sol. The major benefits that can be emanate from One Nation One Ration Card scheme

(ONORC) are: Interoperability of Ration Cards, empowering consumers by giving the opportunity to opt for
the dealer of their choice, reducing social discrimination; particularly for women and other disadvantaged
groups and achieving the Sustainable Development Goals (SDGs) 2030. Therefore, all the above
mentioned options are correct regarding the ONORC.
63. C
Sol. India secured 94th rank among 107 countries of the world in the 'Global Hunger Index 2020' under the
‘serious’ hunger category with a score of 27.2. In the index, India features behind Nepal (73), Pakistan
(88), Bangladesh (75), Indonesia (70) among others. The report also revealed that 14% of India's
population is undernourished. It also says that the country recorded a child stunting rate of 37.4 per cent.
64. C
Sol. The Ministry of Health and Family Welfare has recently launched the 'Intensified Mission Indradhanush
(IMI) 3.0 Scheme' with the aim to cover children and pregnant women who missed routine immunisation
during the Covid-19 pandemic. This scheme will have two rounds in the year 2021 which will be
conducted in 250 pre-identified districts/urban areas across 29 States/UTs. The first edition of this scheme
was launched in October 2017.
65. B
Sol. Food Safety and Standards Authority of India (FSSAI) is a statutory body (established under Food Safety
www.byjusexamprep.com

and Standards Act, 2006) which works under the ambit and jurisdiction of Ministry of Health & Family
Welfare, Government of India. Shri Arun Singhal is currently serving as the Chief Executive Officer
(CEO) of the Food Safety and Standards Authority of India (FSSAI) and Rita Teoatia is the current
Chairperson of FSSAI.
66. A
Sol. X can successfully seek the 'right to shared household' because her relationship with Y falls within the
ambit of "domestic relationship" and that the house in respect of which she seeks to enforce the right is
"shared household".
67. C
Sol. Considering the fact mentioned in the passage, 'Domestic relationship can be through marriage such as
wives, daughters-in-law, sisters-in-law, widows and any other members of the family; or blood relationship
such as mothers, sisters or daughters; and other domestic relationships including through adoption, live-in
relationships, and women in bigamous relationship or victims of legally invalid marriages. Thus, we can
arrive at the answer. Therefore, option C deducing the most appropriate rationale for this reasoning.
68. B
Sol. After analysing the theme of the passage, it is clear that the right to shared household can be enforced by
non-married couples too. In the given scenario, P and Q are non- married couples staying together for a
considerably long time. Right to a shared household can be obtained. Therefore, option B provides the
most appropriate rationale for this reasoning.
69. A
Sol. Here in this case, all the above mentioned legal scenarios are correct regarding the Domestic Violence
Act, 2005 except option A. Domestic violence includes physical violence, sexual violence and mental
violence as well. All these violence can result in the harassment of women by her husband or their
relatives. And any such harassment will be a punishable offence under the court of law. Therefore, option
A is the correct answer for this reasoning.
70. D
Sol. It has been explicitly mentioned in the passage that a woman cannot be thrown out of such a household
except through the procedure established by the law. In case she is thrown out she can be brought back
again after obtaining the order from the court. Hence, there is no such procedure of time limit (12 or 20
years from the marriage) to exit from the house. Therefore, all statements except D are correct in this
case.
71. B
www.byjusexamprep.com

Sol. When the Act was passed in November 2020, the most important concern of the farmers was
the revoking Minimum Selling Price (MSP). It is very understandable, the farmers are
concerned about why the bill is silent on MSP even as the Prime Minister adamantly
guarantees that MSP and floor price will very much be a part of the modified system. It means
that MSP and government procurement will continue the same way as before, and Hari can
sell his tomatoes in local APMC mandi. Therefore, option B provides the most appropriate
rationale for this reasoning.

72. D
Sol. It is clearly mentioned in the passage that the Farmers Produce Trade and Commerce
(Promotion and Facilitation) Bill, 2020 (FPTC) allows farmers to trade outside the mandi and
with private entities without paying any market charges. Hence, Raghav can seek to sell his
products to any private buyer outside the mandi without paying market fee. And he can also
seek to sell his goods across the border of his state/district without any special charges.
Therefore, option D is the correct answer.

73. B
Sol. After analysing the legality and gist of the passage, we can find the evolution of MSP in the
Agricultural Sector. Middleman started exploiting the farmers, they formed cartels or an
understanding among themselves and started buying the product at MSP only and sold to
traders at a high rate; which is the most drastic change that can be observed while comparing
the MSP in 2021 to the 1960s. Therefore, option B provides the most appropriate rationale for
this reasoning.

74. C
Sol. It is clearly mentioned in the last paragraph of the passage that during extraordinary
circumstances like famine, natural calamities, war central government has the power to take
over the regulation of essential commodities. Therefore, the Central government under The
Essential Commodities (Amendment) Bill, 2020 has the right to stock limits if there is a steep
rise in prices. Hence, option C deducing the most appropriate rationale for this reasoning.

75. C
www.byjusexamprep.com

Sol. After analysing the gist of the passage legally and critically, we can clearly deduce that the
Minimum Selling Price (MSP), introduction of corporate/private entities, loss of revenue to the
state, loss of job for middle man; are the correct set of reasons for the concern/ dissatisfaction
of farmers with the introduction of Farm Bills, 2020. Therefore, option C deducing the most
appropriate rationale for this reasoning.

76. B
Sol. The doctrine of frustration applies as the very object of the contract, as recognised by both the
contracting parties, was to have a view of the coronation process. Here in this case, the
happening of the coronation hence forms the heart of the contract. The very object of the
contract was frustrated by the non-happening of the coronation and the plaintiff is held not
entitled to receive money as rent fees. Hence, Krishna is not liable to pay the rent as the
doctrine of frustration applies, the object of contract as recognised by both the parties was to
witness the coronation ceremony and it was impossible. Therefore, option B provides the most
appropriate rationale for this reasoning.

77. A
Sol. In the current scenario, the rise of price by 300% is ridiculously high and is unexpected. It is
impossible to supply the product with the price being 300% higher than at the time of entering
the contract. Thus, X can plead the defence of doctrine of impossibility. Therefore, option A
provides the most appropriate rationale for this reasoning.

78. C
Sol. In the current scenario, Harish can successfully plead frustration. Once he joins the military,
his old employment is automatically terminated thus releasing him from the contract. At the
same time, Harish was no longer a manager and was not bound to the contract in which he
entered earlier. Therefore, option C deducing the most appropriate rationale for this reasoning.

79. C
Sol. It has been mentioned in the passage that Section 32 of the Indian Contract Act, 1872 talks
about contingent contract. Section 32 can be applied only when the contingent event has
happened. In the last paragraph of the passage it is given 'Section 32 only applies when
www.byjusexamprep.com

contracts are discharged and parties absolved of their obligations as per terms already
contained in the relevant contract'. Therefore, option C deducing the most appropriate
rationale for this reasoning.

80. C
Sol. After analysing all the legal scenarios mentioned above, in the 3rd situation, cardiac arrest
puts the risk of health of the defendant making it impossible for him to sing immediately after
an attack. Therefore, the non-coming of singer Zehan is an impossible event under section 56
of the Indian Contract Act 1872. Hence, option C deducing the most appropriate rationale for
this reasoning.

81. B
Sol. Here in this case, both the options A and B are correct answers, but option B gives a more
reasonable explanation. In this case, there was no reasonable apprehension as one could not
assault a person from a moving train. Hence, Bhupesh did not commit assault as there was no
reasonable apprehension of attack nor an imminent threat. Therefore, option B provides the
most appropriate rationale for this reasoning.

82. A
Sol. In the present case, all the necessary requirements of assault are fulfilled. There is an act, a
reasonable apprehension and threat of imminent harm. Hence, Karan will be liable for assault
as all the requirements for assault are fulfilled. Therefore, option A provides the most
appropriate rationale for this reasoning.

83. C
Sol. Here in this case, we can clearly see that matter accelerated and X said he will bring a knife to
kill Y; but he hasn't brought the knife yet. Hence, there was no imminent harm nor a
reasonable apprehension of fear of battery. X was returning towards his house and was not
attacking Y. Hence, there was no assault. Therefore, option C deducing the most appropriate
rationale for this reasoning.

84. B
www.byjusexamprep.com

Sol. It is given in the passage that, in the case of R v. S George, it is said pointing a gun which is
not loaded will also lead to an assault. But it should be at a reasonable distance from where a
person can fear getting injured. Thus, to charge a person with assault it is not necessary that
his act may possibly lead to hurting others. Reasonable apprehension is enough. Therefore,
option B supports the rationale of this reasoning.

85. D
Sol. All of the above mentioned options are correct regarding the elements of the crime of assault.
During assault, while proving a criminal attack, the defendants' behaviour must be motivated
to create a situation of fear or danger in the victim's mind. In the case of assault, the victim
must reasonably believe that the defendant's conduct will harm or humiliate him. The victim's
fear must be a direct response to a threat that is imminent. Therefore, option D provides the
most appropriate rationale for this reasoning.

86. D
Sol. After analysing the theme of the passage, it is clear that the lack of sexual receptivity of a
woman may suggest to a man that the woman is refusing intercourse with him since she may
be indulging in sexual acts with another man outside the marriage. This is a cause of a
husband’s jealousy, which results in him forcing himself upon her to have sexual intercourse.
Therefore, option D provides the most appropriate rationale for this reasoning.

87. B
Sol. After critically analysing the passage, we can clearly deduce that Y has commited marital rape
on his wife in the current scenario. It has been stated in the first paragraph of the passage
about the definition of marital rape. It is unwanted intercourse by a man with his wife by using
force, physical violence or even when she is unable to give consent. In this case too, X was
unable to give consent to her husband as she was intoxicated. Therefore, option B supports
the rationale of this reasoning.

88. C
Sol. It's a very well-known fact that a person cannot be held ‘guilty’ unless he violates any law.
Neither the passage makes marital rape an offence, nor talks about any bill on this issue.
www.byjusexamprep.com

Thus, as per present scenario, Y cannot be guilty of marital rape. Therefore, option C
deducing the most appropriate rationale for this reasoning.

89. A
Sol. It is given in the last paragraph of the passage that the survivor's wife is likely to deal with
bitter feelings of betrayal as the perpetrator may have been someone who was loved and
trusted in the past. Hence, past perpetrators who used to live here, are the most appropriate
reason(s) for the woman raped not to realise that she has been sexually violated and has to
deal with betrayal and broken trust. Therefore, option A provides the most appropriate
rationale for this reasoning.

90. A
Sol. From the last paragraph of the passage, the author makes it aptly clear that there are several
inadequacies in the Indian Penal Code (IPC), lackadaisical attitude of law commissions and
the attitude of ‘male’ legislators. Hence, inadequacies of the law and the attitude of legislators
is the most fundamental reason as to why there is no remedy for marital rape in India.
Therefore, option A provides the most appropriate rationale for this reasoning.

91. C
Sol. It can be clearly inferred from the passage that, there is no legislation to legalise the acts of
euthanasia and physician-assisted suicide. This is clear from the language of the passage in
the third paragraph, that there is no legislation in the country to decriminalize the act of
euthanasia. Therefore, option C deducing the most appropriate rationale for this reasoning.

92. A
Sol. It is clearly understood from the fact stated by the author in the passage that the law is failing
so as to deal with the issue and the relatives of the patient are dealing with the trauma of
losing a loved one. Hence, it is distressing since the law is not equipped at such stage to deal
with the issue of allowing euthanasia. Therefore, option A provides the most appropriate
rationale for this reasoning.

93. D
www.byjusexamprep.com

Sol. Here in the present hypothetical situation, Priya has performed a criminal act. This is clear
from the information given in the passage that so far the action of euthanasia or assisted
suicide has not been decriminalized and therefore would be a criminal act. Hence, option D
would be the most appropriate rationale for this reasoning.

94. C
Sol. After critically analysing the theme of the passage, we can clearly conclude that Priya's act
would still be criminal as consent is not an exception. We can apply the same reasoning as
applied in the previous question because even when consent has been given, it shall fall
under assisted suicide, which has not been decriminalized. Therefore, option C deducing the
most appropriate rationale for this reasoning.

95. C
Sol. This is apparent from a reading of the passage that the author wishes to allow euthanasia
under specific conditions so as to keep the best interest of everyone in mind as is clear from
the third and the fourth paragraph of the passage. Therefore, We must build in checks and
balances so as to allow euthanasia conditionally. Hence, option C deducing the most
appropriate rationale for this reasoning.

96. B
Sol. Here in the current hypothetical situation, Z's adoption is valid and the fact of hiding her real identity from
X's parents does not affect her legal status. It is clearly mentioned in the passage that only two essentials
are required for legal adoption viz. ceremonies and the consent of the wife for a legally valid adoption.
Both were present in this case. Y and X mutually adopted Z and also completed all the legal formalities.
Hence, Z's adoption is a valid one. Therefore, option B provides the most appropriate rationale for this
reasoning.
97. C
Sol. In the present scenario, the adoption of Sourabh is invalid under the law as Suman was not informed
about the same nor her consent was given. In the passage, it is clearly mentioned that the wife has to be
not only informed but her consent is mandatory before adoption by the husband. Satish did not inform
Suman before adopting Sourabh. The fact that there are therapies to correct Suman's problem is
irrelevant here. Therefore, option C deducing the most appropriate rationale for this reasoning.
www.byjusexamprep.com

98. D
Sol. Here in this case, Kavya's adoption is proper and she is now a part of the family. Though Damini was not
willing initially, she later agreed. Hence, the consent of the wife was present. Also it is given that all the
formalities were done properly. Therefore, the adoption is proper. Option (C) is therefore, incorrect. Option
(B) is giving the suggestion outside the scope of the passage. Option (A) is out of context. Therefore,
option D provides the most appropriate rationale for this reasoning.
99. C
Sol. Here in this case, the boy is not an adopted child as the proper procedure for adoption was not followed.
The boy was tricked and separated from his real mother. This is a kind of kidnapping. No proper
procedure, ceremonies or legal formalities were followed. Merely keeping a child at one's home for some
time does not amount to adoption until the formalities and ceremonies are completed. Therefore, option C
deducing the most appropriate rationale which is coherence with the crux of the passage.
100. A
Sol. It is given in the passage that a Hindu adoption is not valid unless the man takes prior consent from his
wife and there is a "ceremony of giving and taking in adoption". It means, only statement I is incorrect
regarding the Adoption procedure under Hindu Law. Therefore, option A provides the most appropriate
rationale for this reasoning.
101. B
Sol. In the current scenario, Mr. X does not have mens rea. In the facts it is clearly given that Mr. X 'was not
aware about the contents of the book'. This implies that he was not conscious that he was selling obscene
material. Hence, he does not have a mens rea. Option (D) is by necessary implication, incorrect. Option
(A) is incorrect as he did have actus reus because he sold the 'obscene' book, though unknowingly, to Mr.
Y. Option (C) is therefore, incorrect. Hence, only option B provides the most appropriate rationale for this
reasoning.
102. A
Sol. Here in this case, Mr. X does not have a mens rea because the day he boarded the flight from Bangkok,
carrying a gold slab was made an offense in India. Actually, he was also carrying the gold which was
within the permissible limit by the Government of India itself and he didn't have a guilty mind while carrying
those gold slabs along with him. Therefore, option A provides the most appropriate rationale for this
reasoning.
103. B
Sol. In the present hypothetical situation, Johnny has not committed a crime as he believed Simran to be 18
www.byjusexamprep.com

years old. Mens rea is the guilty mind. Johnny was told by Simran herself that she is 18 years of age and
Johnny so believed. Hence, Johnny does not have mens rea or the guilty mind. That's why, option (A) is
by necessary implication incorrect. Similarly, options (C) and (D) do not connect the facts to the legal
concepts given in the passage. Hence, option B provides the most appropriate rationale for this reasoning.
104. D
Sol. Here in this case, Sambhav is liable for the death of Shruti as his act implied mens rea in all possible
circumstances. We have to read the facts very carefully. Firstly, it was a busy road. Secondly, Sambhav
was driving at a lightning speed. Thirdly, Sambhav jumps the traffic signals. All these imply gross
negligence and rashness at the end of Sambhav. Naturally, such an act would in all probability sooner or
later result in an accident and may grievously injure a person. Hence, mens rea to cause death or serious
injury can be implied by Sambhav's act as per the reasoning in the second paragraph of the passage.
Unfortunately a pregnant lady, Shruti, became the victim of Sambhav's rashness. Sambhav is therefore
liable for the death of Shruti.
105. D
Sol. All of the above mentioned statements can be legally, logically and critically deprived from the passage.
One of the elements of crime, i.e. mens rea is also implied in the sense that a man is presumed to intend
the natural or probable consequences of his own act. Another important element of crime, Actus Reus can
also be the omission of an act, by omitting to do something that the accused knows he is bound by duty or
law to do. Some crimes do not necessarily require a malafide intention towards one particular person, and
cases involving criminal negligence are a prime example of this. Therefore, option D provides the most
appropriate rationale for this question.
106. C
Sol. Note that the passage throughout talks about the society’s perspective regarding the emotional release. It
mentions that men are much restricted to vent out emotions in public. It also mentions the initiatives being
taken by Japan and other cities to create separate spaces to release their sad and anger emotions. Thus,
only option C tends to convey the comprehensive idea of the passage. Thus, C is the correct answer.
107. D
Sol. The correct answer is option D.

Option A: Note that the ‘feeling rules’ as mentioned in the passage has not been created by any single
person, that can be amended easily. It is a society perspective that cannot be changed in a go. So, this is
not fit to be the correct answer.
www.byjusexamprep.com

Option B: Making public displays of emotions can’t really be made frequent, but yes one can gradually
change his/her own perception related to it, and not consider the showcase of such emotions as
unattractive. This isn’t a correct answer.

Option C: This finds no mention in the passage so this cannot be attributed as the correct answer.

Option D: The passage does mention the creation of separate spaces to help vent emotions of anger and
stress. So, this can be the best way to do it.
108. A
Sol. The correct answer is option A.

Note that the passage states clearly that the value of this kind of service space is evident, especially at a
time when society faces a mental health crisis, with men often worse affected by the inability to talk about
or release their emotions. Thus, option A is the correct answer.

Option B: The statement though is true, but stress can be attributed to mental health, but it nowhere
directly relates to the emotional release spaces and its significance. So, option B is not completely correct
in the given context.

Option C: Japan is just one country that leads on this front. But, the statement mentioned here is
incomplete as it does not specify how it is catering to the growing demand for it.

Option D: The statement just describes the purpose of creating emotional release spaces, but it only
targets customers as mentioned.
109. A
Sol. Option A is the correct answer.

A: if true only then one can attribute feelings and emotions based on time and place as per the ‘feeling
rules’ guide. So, this is correct.

B: Particularly time is not the deciding factors as to when a particular emotion must be felt or showcased.
There are other factors that cause a person to exhibit a particular emotion.

C: This is incorrect. “feeling rules’ is just a sarcasm that the author has used for society’s conventions that
it has for people, their emotions and how they must behave and where.
www.byjusexamprep.com

D talks totally on another tangent so, it is incorrect.


110. C
Sol. Option D reiterates what the author states in the quoted sentence.

Option A: This is an extreme statement, so it is incorrect

Option B: Honoring or dishonoring someone for exhibiting emotions is something out of the context. So,
the option is incorrect.

Option C is the most apt as it states that apologizing is one of the best practices, while in the sentence
author objects as to why most people apologise on seeing someone in tears or themselves if they do it.
So, this statement weakens the given statement.
111. C
Sol. Option A is incorrect as it mentions that the education and research offer no productivity benefits, while the
passage mentions “Much attention is devoted to high-caliber education and research and for good
reason”. Nowhere it undermines productivity but yes it does talk about adding value to it by realizing the
importance of human factors and soft skills.

Option B: This is also untrue as the passage mentions “tendency to focus on cognitive skills, including the
STEM topics of science, technology” which means
112. D
Sol. The correct answer is option D.

Note that the passage through out talks about how not only cognitive skills are necessary, but there is a
dire need to focus on inculcating soft skills that involve human factors. Owing to this, option D is the only
correct choice here.
Option A mentions only cognitive skills which is incorrect.
Option B The passage talks about holistic education and learning new technologies rather than only
concentrating on academics. So this option is also incorrect.
Option C is a fact stated in the passage, it is not just the main theme.
113. A
Sol. The correct answer is A owing to these lines of the passage “The tendency to focus on cognitive skills,
including the STEM topics of science, technology, engineering, and mathematics, cannot preclude the
productivity benefit of soft skills – applying knowledge and finding opportunities offered by technology”
www.byjusexamprep.com

Note that cognitive skill relates to conscious intellectual effort, such as thinking, reasoning, or
remembering. ... Noncognitive or “soft skills” are related to motivation, integrity, and interpersonal
interaction. The author wants to say that much attention is already being given to cognitive skills, but one
needs to realise the true value of soft skills too.

Options B, C and D are true hence cannot be the correct answer here.
114. C
Sol. Note that the author talks about what the current education and research patterns include and focus on.
For it to become holistic and ensure all round development, it must include soft skills that more closely
relate to interpersonal skills so that human improvement aspect can be covered and one can easily adapt
to the required skillset for the plethora of job opportunities knocking the door in the domains of health and
entertainment. So, option C is the correct answer.
115. B
Sol. The correct answer is option B.

Note that A talks about government that is doing wrong by paying attention to high calibre education which
cannot be said outrightly as the statement mentions that focusing on it is done for a good reason. So, it is
false, same is the reason to ride option C incorrect.

Option D is also incorrect as it mentions trends as constant, but reading the given line, it clearly shows
that currenttrends want more focus to be given on human factors, which might have been a case earlier.
116. D
Sol. Note that the passage does mention that the kind of study picked up by Duverger was unique and one of
its own kind because it considered two types of data – election data and survey data to arrive at a
conclusion. But saying that the findings of the study gave an ideal approach mapping the context fo
changing times would be incorrect. The passage does mention that when compared to the findings, the
approach seems to be more durable, but due to the following lines of the passage:
“. Given this failure, Duverger’s study foreshadowed the enduring limitations of the behavioralist approach
to the multinational study of women’s political participation.” We can say that the stated inference is
definitely false.
117. C
Sol. Note that the passage does not put forth the point of inaccuracy of sample data specially because the
study included election and survey data both. In fact, this has been stated as one of the reasons for the
www.byjusexamprep.com

uniqueness of the study. So, I is incorrect.

Statements II mentions adequate expertise which finds no mention in the passage. Statement III is true as
per these lines of the passage “In addition, Duverger’s discussion of his findings was hampered by his
failure to consider certain specific factors important to women’s electoral participation at the time he
collected his data: the influence of political regimes, the effects of economic factors, and the ramifications
of political and social relations between women and men.” So, the correct answer is option C.

Statement IV is irrelevant as there is no mention if the changes could be measured or not.


118. C
Sol. Statement I- True as per this line "Duverger’s research findings were that women voted somewhat less
frequently than men (the difference narrowing the longer women had the vote. and were slightly more
conservative."

Statement II is true as per this line of the passage " However, since these contexts have changed over
time, Duverger’s approach has proved more durable than his actual findings.

Statement III is not completely true. Note that the first half 'women were conservative' is mentioned in first
paragraph. But the latter half of the sentence has been mentioned in the passage as one of the factors
that the author missed to consider which may or may not have influenced the findings. But stating
outrightly that complications existed is not definitely true.

Statement IV is true as the author mentions “(the difference narrowing the longer women had the vote)"
119. B
Sol. The correct answer is option B.

Finding voting patterns is one of the objectives of the study in question and not the article or the passage.

As the passage begins with the mention of the study, the parameters considered and how it is unique, so it
intends to evaluate the research conducted by Duverger. So, option B is the correct answer.

Statement C: There is no trace of reconsidering the findings made by Maurice so this is incorrect.

Statement D: Nowhere the author just talks about political relations between men and women, so this is
just out of the context.
120. C
www.byjusexamprep.com

Sol. The correct answer is option C.

Drawing inference from the given line, it is clear that the gap shortened with the more time allowed to vote.
So, option C becomes the obvious choice.

Statement A: represents an extreme notion so that couldn’t be true.

Statement B: Not much focus has been on the voting pattern of Men, so we cannot say that men used to
exercise their vote to the fullest, so it is also incorrect.
121. B
Sol. Option A can be inferred from the lines mentioned below:

“In particular, India will need to decide whether to join a growing number of countries (over 120 at last
count) that have pledged to reach “net zero” emissions by 2050.”

Hence, it is not the right answer.

Option C can be inferred from the lines mentioned below:

“. Successive Indian governments have taken differing approaches to this in climate negotiations, but one
of the enduring and entirely legitimate planks of our position has been a focus on equity and fairness.”

Hence, it is also not the right answer.

Option D can also be inferred from the first few lines of the passage. Hence, it is rejected.

On the other hand, option B can’t be inferred from the passage at all. Successive Indian Governments
have always focused on achieving climate change goals without compromising on fairness and equity.
But, we can’t consider these to be the pillars on which the idea of net zero emissions is based upon. It has
been stated that the idea of net zero emissions has been inspired by and IPCC report and hence, we can’t
say that it considers fairness and equity as its two pillars. Hence, option B can’t be inferred and hence, it is
the right answer.
122. B
Sol. Statement I can be negated because it goes way too far in its analysis. While it is stated that the initiative
of net zero emission is indeed praiseworthy, we can’t predict that it will certainly bring about positive
www.byjusexamprep.com

reform in our battle against the changing climate. This statement sounds more like someone opinion
rather than something that can be inferred from the passage. Hence, it is not implicit.

Statement III can be negated because it is not mentioned anywhere in the passage.

On the other hand, we can say that statement II is implicit based on the excerpt given below:

“Yet, there are hidden complexities in this formulation. The use of “net” zero potentially allows countries to
keep emitting today while relying on yet-to-be-developed and costly technologies to absorb emissions
tomorrow. Its focus on long-term targets displaces attention from meaningful short-term actions that are
credible and accountable.”

Hence, option B is the right answer.


123. D
Sol. This is perhaps the easiest question of the lot, which can be solved just by going through the last
paragraph of the passage, stated below:

“Yet, there are hidden complexities in this formulation. The use of “net” zero potentially allows countries to
keep emitting today while relying on yet-to-be-developed and costly technologies to absorb emissions
tomorrow. Its focus on long-term targets displaces attention from meaningful short-term actions that are
credible and accountable. And, it calls into question concerns of equity and fairness.”

Hence, option D is the right answer.


124. B
Sol. Let’s eliminate the options one by one:

Options C and D can be easily eliminated because none of them has been mentioned as the possible
challenges that lie in front of us. The author has never really talked about anything that is even remotely
mentioned in these answer option and hence, options C and D are incorrect.

Now, it’s a close call between options A and B. But, if we look closely, we can eliminate option A quite
easily. Option A talks about a particular aspect of what option B is trying to convey to us. That means,
option B encapsulates whatever there is in option A. To verify the same, let’s look at the excerpt given
below:
www.byjusexamprep.com

“Understanding how India can continue to contribute meaningfully to greenhouse gas (GHG) mitigation,
consistent with equity and fairness, is particularly important given a slew of forthcoming climate summits,
at which we will be expected to deliver an updated pledge. In particular, India will need to decide whether
to join a growing number of countries (over 120 at last count) that have pledged to reach “net zero”
emissions by 2050.”

Now, pay attention to whatever is highlighted in bold. The phrase ‘in particular’ means in our bid to
contribute meaningfully to greenhouse gas mitigation, one of the most important aspects is for us to
decide is whether to commit to net zero emissions on not. Hence, we can deduce that whatever there is in
option A, option B consists of the same and then some more. Hence, option B is the right answer.
125. D
Sol. Option A and B can be eliminated just by looking at the first part of the excerpt mentioned above. It is
clearly stated that each country need not reach net-zero emissions by 2050. Hence, options A and B are
incorrect.

Now, the answer to this question lies in one’s ability to figure out the meaning of the phrase “fly in the face
of”. The phrase means to go openly against someone or something. And in option D, it is clearly stated
that countries can go against equity and fairness. Hence, option D is the right answer here.
126. A
Sol.
Refer to these lines from the passage:

“States attaining a high score in 2020 compared to their achievement in 2018, depict much lower
predicted deaths and this relation, though has an inverted u-shape like the previous picture, is sharper and
goes down monotonically for score differences more than 3.5. Thus, investing in sustainable development
goals seems to be helping the states to better manage the pandemic, which is an indicator of
sustainability.”

From the excerpt, we can clearly deduce that the answer to this question in option A. It has been explicitly
mentioned that investing in sustainable development goals has allowed the states to better manage the
pandemic.

Option B is incorrect because it directly contradicts what is stated in the passage.


www.byjusexamprep.com

Options C and D are incorrect because both of them have not been mentioned explicitly in the passage.
The author has not stated anywhere in the passage that sustainable development goals have shifted our
focus from the pandemic. Instead, had all the states focused on a achieving those, our country would have
been in a much better situation.

And even though option D might have been true to some extent in the real world, we can’t predict that
something like that happened if it is not mentioned in the passage.
127. D
Sol. Refer to these lines:

“Though there is discrepancy across states and UTs, the average picture for groups confirms the ability of
high-scorer states/UTs to have been better able to control the pandemic deaths.

This relation becomes all the more pronounced when the 95 per cent confidence interval of predicted
mortality is plotted against the difference in scores of the states between 2018 and 2020.

States attaining a high score in 2020 compared to their achievement in 2018, depict much lower predicted
deaths and this relation, though has an inverted u-shape like the previous picture, is sharper and goes
down monotonically for score differences more than 3.5.”

From the above excerpt, it is evident that option D should be the right answer here. It encapsulates the
main idea of the passage perfectly, while also highlighting the discrepancy associated with the main idea.

Option A is incorrect because while concluding, the author has clearly stated that despite the
discrepancies, a low mortality rate is likely to occur in states with a good SDG score.

Option B, although sounds correct, falls short because of the usage of the conjunction “as soon as”. While
it is true and increase in SDG score can be attributed to lower death rate, it just doesn’t once the score
increases.

Option C is incorrect because it directly contradicts what is stated in the passage. Low COVID-19 mortality
rate is associated with high SDG score and vice-versa. So, an increase in mortality rate would hint
towards decreasing SDG score.
128. B
Sol. Refer to these lines from the passage:
www.byjusexamprep.com

“The Government of India has fasttracked the SDGs and since 2018, annual SDG scores are being
published for different states and Union Territories (UTs) of the country by Niti Aayog. The commitment of
both central and state governments to SDG is clearly visible as the scores show a rising trend for almost
all units in spite of the Covid pandemic challenges. The overall SDG score for India has gone up from 57
in 2018 to 60 in 2019 and to 66 in 2020”

From the above excerpt, it is clear that option B is the right answer here. The year on year stated in the
above-mentioned excerpt is a clear testament to the fact that the government has laid strong emphasis on
the completion of SDG.

Option A is rejected because having a life free from economic and social hardships is practically
impossible. Even in the developed nations, there are people who suffer due to poverty, social
stigmatization and various such issues.

Option C is incorrect because it has not been stated in the passage that various governing bodies has
pressurized the national governments to complete the SDG. Also, the governing bodies that are
supposedly pressurizing the national governments are also not mentioned as well.

Option D is incorrect because it limits the scope of the passage to just “developing nations”. It has to be
kept in mind that the developed nations too focus on achieving the SDG to ensure their status as a
developed nation.
129. C
Sol. Option A is incorrect because it is just the opposite of what we are trying to establish here. If we wish to
establish the importance of achieving SDG, we need to pick a statement that does not diminishes its
value. But here, this answer option s doing the exact opposite by terming the SDG as a burden on the
economy of the states.

Option B is incorrect because we don’t know anything about other parameters that can be used to ensure
low mortality rates. Also, if we talk about the prominence of having other parameters in lowering the
mortality rates, we are diminishing the influence of the SDG, which need not be done here.

Option D is incorrect because even though it terms achieving SDG a worthy feat, it also talks about the
process being tedious.
www.byjusexamprep.com

Option C is the correct answer here because it only tries to strengthen the case of achieving SDG by
pointing out the results of a report which highlights the prominence of SDG.
130. A
Sol. Ans. Let’s go by option elimination here:

Option B is incorrect because it is way too extreme in its analysis. Nowhere in the passage it is mentioned
that achieving the SDGs should be prioritized at all costs.

Option C is incorrect because it is slightly missing the true purpose of the SDG. The SDGs encompass
different aspects of life, factoring in economical, ecological and social aspects as well. However, the
answer option does not factor in the social aspects of the SDGs.

Option D is incorrect because it sounds more like an opinion of a person, something which is not done
explicitly in the passage. The author has used data and facts to point out the prominence of SDGs but
nowhere in the passage it is mentioned that the author thinks SDGs are a great way to gauge mortality
rates associated with the pandemic. Besides, vaccination rates are not discussed anywhere in the
passage.
131. A
Sol. The whole passage has talked about the miscalculation of the number of actual casualties related to the
pandemic. By focusing on the same, the author has stressed out that there is a pressing need to carry out
a nationwide demographic survey. Now, if we wish to strengthen this claim, we need to pick a statement
that talks along the same lines.

Option B is rejected because if we say that the government has succeeded partially in its attempt to
calculate an exact number of death, then the whole purpose of having a demographic survey gets
weakened.

Option C is straightaway denying the need for conducting a nationwide demographic, thereby weakening
the argument.

Option D is incorrect because even though it talks about the government considering the need to push for
the survey, it does not take into account the reason why it should be done.

Option A is the right answer because it clearly specifies the reason why we need to have a demographic
survey and hence, it is the right answer.
www.byjusexamprep.com

132. A
Sol. If we need to weaken the claim that there is a need to perform a nationwide demographic survey, we need
to pick a statement that highlights the downsides of conducting such a survey.

Option B is incorrect because it talks about a potential advantage of conducting such an experiment.

Option C is incorrect because what happened in a survey in the U.S has no clear correlation with the
situation in India. And just because the survey was conducted successfully in the U.S, it does not mean it
will be conducted successfully in India as well.

Option D is incorrect because the government’s intentions have nothing to do with the need for carrying
out such an experiment.

Option A is correct because it takes into account a potential downside of conducting such a survey.
Hence, it is the right answer.
133. C
Sol. Here, it’s better to go by option elimination straightaway:

Option A is incorrect because we can’t say that the government has always shied away from the
negativity surrounding the deaths. The answer option is too extreme to even consider and besides, it
doesn’t explain what is stated in the excerpt mentioned above.

Option B is incorrect because it sounds like a corrective measure that needs to be followed to get to a
particular solution, which is not the case here.

Option D is incorrect because this is not stated anywhere in passage, let alone in the excerpt mentioned in
the question.

Option C is the right answer here because it clearly explains what is mentioned in the lines stated in the
question. The question has talked about the incentives associated with undercounting the numbers of
deaths and option C has explained the exact same thing to us.
134. A
Sol. This question can easily be answered by looking at the answer options closely. Of all the answer options,
option A is definitely not true here. Nowhere in the passage it is mentioned that the Government of India is
deliberately trying to undercount the number of casualties associated with the pandemic. Although there
www.byjusexamprep.com

has been misreporting of cases and death in the nation, we can’t say that the Union Government has got
a hand in this. This answer options puts the blame on the Central Government’s shoulders, something that
is not done by the author. Hence, this is a false statement and hence, it is the right answer.
135. D
Sol. From the context, it is clear that we just can’t establish the relationship between the accuracy at which
both these systems work. For some, CRS works best while others believe in SRS. This can be further
highlighted by the excerpt given below:

“Arvind Subramanian, a former GoI chief economic advisor, and his associates, recently estimated that,
under some assumptions, using CRS shows that excess mortality between April 2020 and June 2021 was
3.4 million. There’s an argument against using CRS, which is that it’s not as accurate as GoI’s SRS. That’s
partly true. Data of 2019 shows that in the southern states there’s no mismatch between CRS and SRS.
But many other states’ CRS underestimate deaths.”

Hence, option D is the right answer.


136. B
Sol. Total no. of surveyed persons who gave suggestion of only social distancing = 1600

Total no. of surveyed persons who gave suggestion of only wearing a mask = 1600

Total no. of surveyed persons who gave suggestion of only using sanitizer = 1600

Total no. of surveyed persons who gave suggestion of only washing hands with anti – biotic soap = 1600

Total no. of surveyed persons who gave suggestion of vaccine only =

Total no. of surveyed persons who gave suggestion of only social distancing and wearing a mask both =
1600

Total town no. of surveyed persons who gave suggestion of only social distancing, wearing a mask and
vaccine all =
www.byjusexamprep.com

Total no. of surveyed persons who gave suggestion of only wearing a mask, washing hands with soap and
vaccine all =

Total no. of surveyed persons who gave all the five suggestions = 00

Total people out of the surveyed people who gave suggestion of using Sanitizer only, washing hands with
anti – biotic soaps only and vaccine together = 192 + 128 + 320

= 640

Total people out of the surveyed people who gave suggestion of only social distancing & only wearing a
mask together

= 400 + 160

=560

Required percentage =

= 14.28%
137. D
www.byjusexamprep.com

Sol. Total no. of surveyed persons who gave suggestion of only social distancing = 1600

Total no. of surveyed persons who gave suggestion of only wearing a mask = 1600

Total no. of surveyed persons who gave suggestion of only using sanitizer = 1600

Total no. of surveyed persons who gave suggestion of only washing hands with anti – biotic soap = 1600

Total no. of surveyed persons who gave suggestion of vaccine only =

Total no. of surveyed persons who gave suggestion of only social distancing and wearing a mask both =
1600

Total town no. of surveyed persons who gave suggestion of only social distancing, wearing a mask and
vaccine all =

Total no. of surveyed persons who gave suggestion of only wearing a mask, washing hands with soap and
vaccine all =

Total no. of surveyed persons who gave all the five suggestions = 00
www.byjusexamprep.com

Total no. of people out of surveyed people who gave at most two suggestions

= 400 + 160 + 192 +128 + 320 + 128

= 1328
138. D
Sol. Total no. of surveyed persons who gave suggestion of only social distancing = 1600

Total no. of surveyed persons who gave suggestion of only wearing a mask = 1600

Total no. of surveyed persons who gave suggestion of only using sanitizer = 1600

Total no. of surveyed persons who gave suggestion of only washing hands with anti – biotic soap = 1600

Total no. of surveyed persons who gave suggestion of vaccine only =

Total no. of surveyed persons who gave suggestion of only social distancing and wearing a mask both =
1600
www.byjusexamprep.com

Total town no. of surveyed persons who gave suggestion of only social distancing, wearing a mask and
vaccine all =

Total no. of surveyed persons who gave suggestion of only wearing a mask, washing hands with soap and
vaccine all =

Total no. of surveyed persons who gave all the five suggestions = 00

Total no. of people out of surveyed people who gave at least two suggestions

= 128 + 96 + 80 +96

= 400
139. D
Sol. Total no. of surveyed persons who gave suggestion of only social distancing = 1600

Total no. of surveyed persons who gave suggestion of only wearing a mask = 1600

Total no. of surveyed persons who gave suggestion of only using sanitizer = 1600
www.byjusexamprep.com

Total no. of surveyed persons who gave suggestion of only washing hands with anti – biotic soap = 1600

Total no. of surveyed persons who gave suggestion of vaccine only =

Total no. of surveyed persons who gave suggestion of only social distancing and wearing a mask both =
1600

Total town no. of surveyed persons who gave suggestion of only social distancing, wearing a mask and
vaccine all =

Total no. of surveyed persons who gave suggestion of only wearing a mask, washing hands with soap and
vaccine all =

Total no. of surveyed persons who gave all the five suggestions = 00

Required ratio =

=5:6
www.byjusexamprep.com

140. D
Sol. Total no. of surveyed persons who gave suggestion of only social distancing = 1600

Total no. of surveyed persons who gave suggestion of only wearing a mask = 1600

Total no. of surveyed persons who gave suggestion of only using sanitizer = 1600

Total no. of surveyed persons who gave suggestion of only washing hands with anti – biotic soap = 1600

Total no. of surveyed persons who gave suggestion of vaccine only =

Total no. of surveyed persons who gave suggestion of only social distancing and wearing a mask both =
1600

Total town no. of surveyed persons who gave suggestion of only social distancing, wearing a mask and
vaccine all =

Total no. of surveyed persons who gave suggestion of only wearing a mask, washing hands with soap and
vaccine all =

Total no. of surveyed persons who gave all the five suggestions = 00
www.byjusexamprep.com

Required percentage =

= %
141. B
Sol. We will find the required data by given information.

Cost of a mouse= 1000

Cost of a printer= 5 1000= 5000

Cost of a Ipad= 3 5000= 15000

Cost of a scanner= 15000= 9000

Cost of a CPU= 2000 + 15000= 17000

Cost of a monitor= 15000= 19500

Cost of a laptop= 5 9000= 45000


www.byjusexamprep.com

Playstation=

Required profit= .
142. A
Sol. We will find the required data by given information.

Cost of a mouse= 1000

Cost of a printer= 5 1000= 5000

Cost of a Ipad= 3 5000= 15000

Cost of a scanner= 15000= 9000

Cost of a CPU= 2000 + 15000= 17000

Cost of a monitor= 15000= 19500

Cost of a laptop= 5 9000= 45000

Playstation=

Required total cost= 2 16000 + 1 45000 + 3 9000= 104000.


143. D
Sol. We will find the required data by given information.

Cost of a mouse= 1000

Cost of a printer= 5 1000= 5000

Cost of a Ipad= 3 5000= 15000

Cost of a scanner= 15000= 9000

Cost of a CPU= 2000 + 15000= 17000


www.byjusexamprep.com

Cost of a monitor= 15000= 19500

Cost of a laptop= 5 9000= 45000

Playstation=

Required ratio= 9000 : 19500= 6 : 13.


144. B
Sol. We will find the required data by given information.

Cost of a mouse= 1000

Cost of a printer= 5 1000= 5000

Cost of a Ipad= 3 5000= 15000

Cost of a scanner= 15000= 9000

Cost of a CPU= 2000 + 15000= 17000

Cost of a monitor= 15000= 19500

Cost of a laptop= 5 9000= 45000

Playstation=

Cost of laptop= 45000

Discount= 10%

Therefore C.P= 0= 40500

Now M.P of laptop= 40500= 48600

Discount given by Anup on laptop= 5%


www.byjusexamprep.com

Therefore, S.P of laptop= 48600= 46170

Required profit %=

=
145. A
Sol. We will find the required data by given information.

Cost of a mouse= 1000

Cost of a printer= 5 1000= 5000

Cost of a Ipad= 3 5000= 15000

Cost of a scanner= 15000= 9000

Cost of a CPU= 2000 + 15000= 17000

Cost of a monitor= 15000= 19500

Cost of a laptop= 5 9000= 45000

Playstation=

Required percentage=
146. A
Sol. Let total Non-electronic & Electronic sold by Myntra be ‘40x’

So, total Non-electronic & Electronic sold by Flipkart = 40x

= 50x

And, total number of Non-electronic & Electronic sold by Amazon = 3800 × 2 – 40x
www.byjusexamprep.com

= 7600 – 40x

Now, total number of Non-electronic & Electronic sold by Ajio

ATQ,

⇒ 2280 – 12x – 8x = 280

⇒ 20x = 2000

⇒ x = 100

Required percentage

= 50%
147. B
Sol. Let total Non-electronic & Electronic sold by Myntra be ‘40x’

So, total Non-electronic & Electronic sold by Flipkart = 40x

= 50x

And, total number of Non-electronic & Electronic sold by Amazon = 3800 × 2 – 40x

= 7600 – 40x

Now, total number of Non-electronic & Electronic sold by Ajio


www.byjusexamprep.com

ATQ,

⇒ 2280 – 12x – 8x = 280

⇒ 20x = 2000

⇒ x = 100

Electronic items sold by all four companies = (3600 – 1080) + (5000 – 800) + (4000- 800) + (3000 – 1200)

= 2520 + 4200 + 3200 + 1800

= 11720

Required average = 2930


148. D
Sol. Let total Non-electronic & Electronic sold by Myntra be ‘40x’

So, total Non-electronic & Electronic sold by Flipkart = 40x

= 50x

And, total number of Non-electronic & Electronic sold by Amazon = 3800 × 2 – 40x

= 7600 – 40x

Now, total number of Non-electronic & Electronic sold by Ajio


www.byjusexamprep.com

ATQ,

⇒ 2280 – 12x – 8x = 280

⇒ 20x = 2000

⇒ x = 100

Non-electronic sold by Amazon & Ajio together = 1080 + 1200 =2280

Electronic sold by Ajio = 1800

Required % more
149. D
Sol. Let total Non-electronic & Electronic sold by Myntra be ‘40x’

So, total Non-electronic & Electronic sold by Flipkart = 40x

= 50x

And, total number of Non-electronic & Electronic sold by Amazon = 3800 × 2 – 40x

= 7600 – 40x

Now, total number of Non-electronic & Electronic sold by Ajio

ATQ,
www.byjusexamprep.com

⇒ 2280 – 12x – 8x = 280

⇒ 20x = 2000

⇒ x = 100

Required ratio

150. A
Sol. Let total Non-electronic & Electronic sold by Myntra be ‘40x’

So, total Non-electronic & Electronic sold by Flipkart = 40x

= 50x

And, total number of Non-electronic & Electronic sold by Amazon = 3800 × 2 – 40x

= 7600 – 40x

Now, total number of Non-electronic & Electronic sold by Ajio

ATQ,

⇒ 2280 – 12x – 8x = 280

⇒ 20x = 2000
www.byjusexamprep.com

⇒ x = 100

Total Non-electronic sold by all four companies = [1080 + 800 + 800 + 1200]

= 3880

Average of total Non-electronic & Electronic sold by Flipkart, Myntra & Ajio = 4000

Required difference = 4000 – 3880 = 120

You might also like